Professional Responsibility Multiple Choice

Ace your homework & exams now with Quizwiz!

In Attorney's closing statement to the court in a bench trial, Attorney said, "Your honor, I drive on the street in question every day and I know that a driver cannot see cars backing out of driveways as the one did in this case. I believe that my client was not negligent, and I ask you to so find." Was Attorney's closing argument proper? a. Yes, because the rules of evidence are very liberal when the trial is before a judge without a jury. b. Yes, if Attorney was speaking truthfully and not trying to deceive the court. c. No, because Attorney asserted Attorney's personal knowledge of facts in issue. d. No, if there is no other evidence in the record about the facts asserted by Attorney.

C.

Client Chason hired lawyer Lucero to do the legal work in connection with a complex public securities offering. Lucero agreed to do the work for $160 per hour. Lucero did a great deal of legal research, prepared numerous memoranda of fact and law, and drafted most of the documents needed for the public offering. At that point, Chason became angry with Lucero for no apparent mason and fired him. Chason paid Lucero at the agreed rate for the work Lucero had done, and Chason demanded that Lucero turn over to him the papers that Lucero had prepared, including the legal and fact memoranda and the document drafts. What papers must Lucero turn over to Chason? a. Only the document drafts, but not the legal and fact memoranda. b. Only the legal and fact memoranda, but not the document drafts. c. None of the papers, because Chason fired Lucero. d. All of the papers, even though Chason fired Lucero.

A.

A series of brutal daylight muggings in downtown Sedatia brought fear to the citizens of that normally placid city. The police captured one Diablo, who was charged with the muggings and, in due course, was ordered to stand trial in Sedatia. Two days before the jury selection began, a local newspaper reporter cornered the prosecutor, District Attorney Axelrod, in the Sedatia Cafe and got her to make the following statement: I'm certain Diablo is the right man; among other things, we have discovered that he was previously convicted three times for brutal muggings in other states. Is Axelrod subject to discipline for making the statement to the reporter? a. No, because a lawyer has a First Amendment right to inform the public about pending cases. b. Yes, because she should have known that the statement would be quite likely to prejudice the trial. c. Yes, because a prosecutor must not make public comment on a pending case. d. No, because prior criminal convictions are a matter of public record.

B.

Able, Baker and Charley have a video production company. It is organized pursuant to a written partnership agreement, with shares of 40%, 40% and 20%. The 3 partners have reached an impasse concerning a revision of their agreement. Charley is the active manager, contributing time, energy and enthusiasm, while Able and Baker are more passive investors. Charley desires a greater partnership share. Able and Baker are content with the present agreement. You have, for some time, represented Able in various real estate ventures. Those ventures are unrelated to the video company. The three partners come to you for legal help. They ask you to assist them in reaching an agreement and putting it into writing. You tell Baker & Charley that you represent Able in other matters. May you represent all three partners? A) No, because their interests are directly adverse to each other. B) Yes, provided all three consent after full consultation and disclosure of the implications of the common representation. C) No, because you represent Able in other matters. D) Yes, provided all three agree to be bound by your ultimate recommendations. E) No, because lawyers are advocates and may not function as mediators.

B.

According to the Code of Judicial Conduct, which of the following activities are improper for a judge? I. Teaching a course in advanced trial practice at a seminar for practicing attorneys, for a fee. II. Soliciting funds from the public to support the National Council on Penal Reform. III. Writing an article for a national magazine on fishing in the Great Lakes region. IV. Serving as the weekend manager at a local antique store. a. All of the above. b. II. and IV. only. c. I., II., and IV. only. d. I. and III. only.

B.

After lawyer Laben graduated from law school, she joined the congressional staff of U.S. Senator Senders. In that role, she personally drafted a bill that was ultimately enacted as the Educational Rights of Disabled Americans Act ("ERDAA"), a far-reaching statute that required colleges and universities to make many changes in the facilities they supply to blind, deaf, and physically-impaired students. Shortly thereafter, Laben moved to State A, where she became an associate in the private law firm of Dillard & Domish. Seeking legal advice on how to comply with ERDAA, the University of State A hired partner Dillard and specifically asked Dillard to assign Laben to assist him in doing the work. In light of Laben' s earlier role as the drafter of ERDAA, which of the two lawyers may work on the matter? a. Neither Dillard nor Laben. b. Both Dillard and Laben. c. Dillard only, and only if Laben is properly screened off from the matter. d. Dillard only, and only if the University consents after full disclosure.

B.

Agnes and Bertram represent two corporations who oppose each other in a civil suit. Agnes has filed a petition with the court, seeking to have Bertram removed as opposing counsel. She claims he has a conflict of interest since he once did certain work for her present client. Bertram calls Agnes and tells her that he thinks the conflict of interest matter can be resolved if she listens to his explanation. Agnes and Bertram meet and begin to negotiate the conflict of interest issue, and during the same meeting discuss a settlement of the lawsuit. Bertram and Agnes agree to meet again and continue their discussions. Immediately alter the first meeting, Agnes goes to her office and dictates a letter to Judge Jowly, who is trying the case. The letter tells Jowly that Agnes and Bertram arc attempting to resolve their differences on the conflict of interest matter and are also negotiating a settlement of the underlying lawsuit. Agnes mails the letter to Jowly that afternoon. Is Agnes subject to discipline? a. No, if there were no false or prejudicial statements in the letter b. Yes, unless Agnes sends Bertram a copy of the letter c. Yes, because Agnes failed to inform Bertram before she sent the letter. d. No, because it is courteous to inform the judge that a settlement is being negotiated.

B.

Alpha represents Defendant in bitter and protracted litigation. Alpha, at Defendant's request, has made several offers of settlement to Plaintiff's lawyer Beta, all of which have been rejected. During a one-week recess in the trial, Alpha and Plaintiff were both present at a cocktail party. Plaintiff went over to Alpha and said: "Why can't we settle that case for $50,000? This trial is costing both sides more than it's worth." Which of the following is a proper response by Alpha? I. "I can't discuss the matter with you." II. "If that's the way you feel, why don't you and Defendant get together." III. "I agree. We already have made several offers to settle this matter." a. II and III, but not I b. I only c. I and II, but not III d. I, I1, and II

B.

Attorney Adlon and his long term client, Chuckles, had a very close personal relationship that went far beyond the lawyer-client association. They often took fishing trips in each other's company, played golf together, and sometimes shared "bachelor dinners" after the deaths of their respective spouses. Thus, Adlon was deeply saddened when Chuckles's daughter Doris came to his office and suggested that Chuckles had become incompetent to handle his affairs. Adlon had to agree with Doris, because Adlon could not help but notice that Chuckles's mind often wandered, and that sometimes Chuckles thought that he and Adlon were still back in high school together. Although one of Chuckles's sons, Sam, opposed Doris's attempt to have Chuckles declared incompetent, Adlon agreed to handle the case and Adlon's arguments prevailed at the competency hearing. Chuckles was declared incompetent, and Tommy Trustee was placed in control of Chuckles's finances. A month after the competency hearing, Chuckles's other son, Sidney, came into Adlon's offices complaining that Chuckles had given Doris a valuable gift several months previously, and Sidney asked Adlon to file suit to have the gill set aside. Because of Adlon's intimate involvement in Chuckles's affairs, it is clear that Adlon would have to testify during the trial of any such suit. May Adlon represent Sidney? a. Yes, provided that he associates with another attorney who will do the cross-examination when Adlon testifies and the client approves the association in advance. b. No, because it is certain that Adlon will have to testify at the trial as to substantive matters. c. Yes, because Adlon was so involved in Chuckles's affairs that he is probably better able to handle tile suit than any other attorney. d. No, because Adlon previously sided with Doris in the competency hearing.

B.

Attorney Aggie is prosecuting a complex tax case. After two government witnesses testified, the defendant, Daft, took the stand in his own defense. Daft asserted that he always complied with all tax rules and regulations. He also testified that the two government witnesses had deliberately falsified Daft's Internal Revenue Service records because the witnesses belonged to a Satanic cult which bore a long-standing grudge against Daft. In Aggie's closing argument, she made the following statements relative to the defendant's testimony: I. "Defendant's testimony is clearly in conflict with the testimony of two government witnesses." II. "Of the persons who have given testimony, who has the better reason to lie, the defendant or the government's witnesses?" III. "If you believe the testimony of the defendant, you will also believe the Moon is made of green cheese!" For which, if any, of these statements would Aggie be subject to discipline'? a. I., II., and III. b. III. only. c. 1I. and III. only. d. Neither l., II., nor III.

B.

Attorney Aimas was fully licensed to practice in State Green. tier office wits located in Endogreen, a small town located on the extreme western border of State Green, on the western edge of a desert area. Aimas received a retainer /Yom client Chum, with the agreement that Alinas would use funds from the retainer for such things as filing fees when they came due. There was no federally insured bank or savings institution in Endogreen. The nearest such institution in State Green was located in the county seat, which was over 60 miles away from Alma's office and where Aimas and Chum resided. Therefore, Almas decided to put Chum's money in his client trust account in a bank in Bluestart, a medium-sized city located just across the state line in neighboring State Blue. The account was fully insured by the federal government, but was not an interest bearing account. Almas is not licensed to practice law in State Blue. Was it proper fur Aimas to place Chum's money in an account in State Blue? a. Yes, because client funds must be placed in a safe account and the closest federally insured institution was in Bluestart. b. No, because Aimas is not licensed to practice in State Blue. c. No, because the funds were not placed in an interest bearing account. d. Yes, because retainer fees belong to tile attorney and not to the client.

B.

Attorney Alexander and her client Cardone endured a stormy attorney-client relationship until Alexander finally withdrew due to Cardone's repeated refusals to pay Alexander's fee bills. At the end of the relationship, Cardone owed Alexander more than $10,000. Cardone said he would not pay because Alexander's legal services were "defective." In a final effort to avoid having to sue Cardone for the unpaid fees, Alexander proposed a settlement agreement to Cardone. Under the proposed agreement, Alexander would accept $4,000 as full payment, reserving the right to sue Cardone for the other $6,000 if Cardone filed a State Bar disciplinary complaint against Alexander or filed a legal malpractice action against Alexander. Cardone signed the settlement agreement without consulting outside counsel, and Alexander did not suggest that he should consult outside counsel before signing it. Is Alexander subject to discipline for entering into the settlement agreement with Cardone? a. Yes, because Alexander compromised a potential malpractice claim by contract with her client. b. Yes, because Alexander did not advise Cardone to seek outside counsel before entering into the settlement agreement. c. No, because Alexander brought about an amicable settlement of the fee dispute with Cardone. d. No, provided that there was a good faith dispute between Alexander and Cardone about the quality of Alexander's services and the amount of fees due.

B.

Attorney Alpha has been employed as an assistant prosecutor in the District Attorney's office during the time that an investigation of Deft was being conducted by that office. Alpha took no part in the investigation and had no knowledge of the facts other than those disclosed in the press. Two months ago, Alpha left the district attorney's office and formed a partnership with Attorney Beta. Last week, Deft was indicted for offenses allegedly disclosed by the prior investigation. Deft asked Alpha to represent him. Alpha declined to do so, but suggested Beta. Is Beta subject to discipline if Beta represents Deft? a. Yes, because Alpha was employed in the district attorney's office while the investigation of Deft was being conducted. b. No, because Alpha had no responsibility for or knowledge of the facts of the investigation of Deft. c. Yes, unless the District Attorney's office is promptly notified and consents to the representation. d. No, unless Alpha participates in the representation or shares in the fee.

B.

Law professor Pompman was selected as the neutral arbitrator of a boundary line dispute between land owners Owens and Osborne. Pompman decided the matter in favor of Owens. Shortly thereafter, Pompman quit his teaching position and entered private law practice. Osborne brought suit to have the arbitration award set aside. Owens asked Pompman to represent him in the suit. If Pompman takes the case, will he be subject to discipline? a. Yes, because there is reasonable ground to doubt his impartiality in the case. b. No, because by seeking to hire Pompman, Owens is deemed to have consented to the conflict of interest c. Yes, because his earlier service as neutral arbitrator creates a conflict of interest. d. No, because serving as Owens's lawyer is consistent with his decision as arbitrator in favor of Owens.

C.

Attorney Alpha represents Wife in a marriage dissolution proceeding that involves bitterly contested issues of property division and child custody. Husband is represented by Attorney Beta. After one day of trial, Husband, through Beta, made a settlement offer. Because of Husband's intense dislike for Alpha, the proposed settlement requires that Alpha agree not to represent Wife in any subsequent proceeding, brought by either party, to modify or enforce-the provisions of the decree. Wife wants to accept the offer, and Alpha believes that the settlement offer made by Husband is better than any award Wife would get if the case went to judgment. Is it proper for Alpha to agree that Alpha will not represent Wife in any subsequent proceeding? a. Yes, because the restriction on Alpha is limited to subsequent proceedings in the same matter. b. No, because the proposed settlement would restrict Alpha's right to represent Wife in the future. c. No, unless Alpha believes that Wife's interests can be adequately protected by another lawyer in the future. d. Yes, if Alpha believes that it is in Wife's best interests to accept the proposed settlement.

B.

Attorney Alpheus worked for two years for the Veteran's Administration. While there, his main function was to investigate claims filed by veterans. During the course of his employment, he once investigated a claim filed by Charles, a Vietnam War veteran. After Alpheus left the Veteran's Administration, the agency denied Charles's claim. Charles comes to Alpheus, who is now engaged in private practice, and asks him to represent him in a suit against the Veteran's Administration for the benefits to which Charles believes he is entitled. Is Alpheus subject to discipline if he accepts Charles's case? a. No, if Alpheus was not privy to confidential information regarding Charles, arising from his employment at the Veteran's Administration. b. Yes, if Alpheus had substantial and personal responsibility for Charles's Veteran's Administration claim. c. No, because Alpheus has left the Veteran's Administration. d. Yes, because Alpheus had at least some knowledge of Charles's claim when Alpheus was employed by a government agency.

B.

Attorney Anthony Altamirez has organized his law practice as a professional corporation. Altamirez is the sole shareholder. The sign on the office door states: Anthony Ahamirez, P.C.-- Attorney at Law/Corporate and Business law Torts and Domestic Relations...Altamirez has only one lawyer-employee, Leola Lipkis, who was admitted to practice two years ago. Altamirez pays Lipkis a modest monthly salary plus 60% of the fees collected in cases that Lipkis handles by herself. Altamirez has a general business practice, and when a client needs representation in a tort or domestic relations matter, Altamirez turns the case over to Lipkis. When Altamirez turns a case over to Lipkis, he provides general guidance and is available to answer any questions she may have, but he does not supervise every step she takes. Is Altamirez subject to discipline? a. No, unless he is not certified as a specialist in the areas noted on his office sign. b. No, because Lipkis is a lawyer-employee of Altamirez. c. Yes, because he splits fees with Lipkis in matters she handles by herself. d. Yes, because he does not closely supervise the work done by Lipkis.

B.

Attorney April represents Chuck, plaintiff in a personal injury suit arising out of an automobile collision. Chuck asserts that he had the right-of-way to enter the intersection where the accident occurred. The defendant claims otherwise, but Chuck tells April that there was a witness present who would be able to verify Chuck's version of the accident. April obtains a copy of the police report on the accident, but the name of the witness is not contained in the report. April contemplates running an ad in the newspaper or hiring an investigator to find the witness, but April concludes that Chuck's testimony ought to be strong enough to win the case. The case comes to trial, and the jury finds for the defendant. Is April subject to discipline for failure to try to find the witness? Is Lubner's conduct proper? a. Yes, because her client lost. b. No, if she reasonably believed that Chuck's testimony would be sufficient. c. Yes, because she failed to properly prepare the case. d. No, because the name of the witness was not in the police report.

B.

Attorney Arlo has hired Clarissa, a third year student at a local law school, to assist him as his clerk. Clarissa is not licensed under any state law or court rule that allows third-year law students to engage in practice under the supervision of a licensed attorney. Arlo has Clarissa perform the following tasks: I. Drafting a release form for personal injury plaintiffs to sign after their cases have been settled. (Arlo himself has the plaintiffs sign the forms.) II. Interviewing witnesses to accidents, and having them sign Clarissa's written version of the interview. III. Reaching settlement agreements with insurance company representatives before suit has been filed. For which, if any, of the above is Arlo subject to discipline? a. I. only. b. III only. c. I. and 1I., but not III d. II. and III, but not I.

B.

Attorney Dave is defending Datatec Corporation in an employment discrimination suit in which the plaintiffs are represented by attorney Perclio. Wallner is a Datatec employee, but he is not an officer or shareholder in Datatec, and he is not a party to the lawsuit. Wallner is in charge of Datatec's Personnel Department, and he is responsible for insuring that Datatec's hiring practices comply with the laws against employment discrimination. Wallner is in poor health, so Dave scheduled the taking of Wallner's deposition as a precaution in case Wallner should die be fore trial. Without seeking Dave's consent, or even telling him, Perello had lunch with Wallner several days before the deposition, and on that occasion Perello pumped Wallner for information relevant to the lawsuit. When Dave learned what had happened, he telephoned Perello and called him a slimy, mud-sucking shyster. Which of the following is most nearly correct? a. Perello is subject to discipline because he talked with a deposition witness about the subject of the litigation before the deposition was taken. b. Perello is subject to discipline because he should not have talked with Wallner about the case without Dave's consent. c. Dave's conduct was proper. d. Perello's conduct was proper.

B.

Defendant Duncan hired lawyer Lamar to defend him at his trial for second degree murder. The day before the trial, Duncan asked Lamar if he could tell Lamar something in complete confidence. Lainar said he could. Duncan said: "The truth is that my son, Samuel, is the one who committed the murder. He's a good boy, and I'm willing to go to jail to protect him. I will not testify at the trial, and I don't want you to put on any false evidence. Do the best you can to defend me, but don't do anything that would point the finger at Samuel." Lamar discussed all of the possible alternatives with Duncan, but Duncan refused to change his mind. What should Lamar do'? a. Withdraw as Duncan's lawyer, and advise Duncan not to tell his new lawyer about Samuel's involvement. b. Keep Duncan's statement in confidence, and defend Duncan as best he can. c. Reveal Duncan's statement to the trial judge, and let the trial .judge decide what to do. d. Advise Duncan to change his plea to guilty, and prepare to argue for a lenient sentence.

B.

Attorney is a lawyer for City and advises City on all tort claims filed against it. Attorney's advice is limited to recommending settlement and the amount thereof. If a claim is not settled and suit is filed, defense of the suit is handled either by lawyers for City's insurance carrier or by outside counsel specially retained for that purpose. In connection with any notice of claim and before suit is filed, Attorney arranges for an investigator to call upon the claimant at the claimant's home and, with no one else present, to interview the claimant and endeavor to obtain a signed statement of the claimant's version of the facts. Claimant has filed a notice of claim against City. Attorney has sent an investigator to interview Claimant. Is Attorney subject to discipline for arranging an interview with Claimant? a. No, because claimant had not filed suit at the time of the interview. b. Yes, if Claimant was known by Attorney to be represented by counsel. c. No, because Attorney would not be representing City in any subsequent litigation on Claimant's claim. d. Yes, if the statement taken is later used to Claimant's disadvantage.

B.

Attorney is a we/l-known, highly skilled litigator. Attorney's practice is in an area of law in which the trial proceedings are heard by the court without a jury. In an interview with a prospective client, Attorney said, "I make certain that I give the campaign committee of every candidate for elective judicial office more money than any other lawyer gives, whether it's $500 or $5,000. Judges know who helped them get elected." The prospective client did not retain Attorney. Is Attorney subject to discipline? a. No, if Attorney's statements were true. b. Yes, because Attorney implied that Attorney receives favored treatment by judges. c. Yes, if Attorney's contributions are made without consideration of candidates' merits. d. No, because the prospective client did not retain Attorney.

B.

Attorneys A and B are law partners. They are, however, contemplating incorporation of their law practice. With respect to the proposed incorporation, which of the following statements best describes what A and B may properly do? I. A and B may incorporate their law practice and convey an interest in the corporation to their children. II.. A and B may incorporate their law practice and thus avoid liability to clients for malpractice. III. A and B may incorporate their law practice and when they die, a fiduciary representative of their estates may hold the stock in the corporation for a reasonable time during administration. IV. A and B may incorporate their law practice and make their accountant treasurer of the corporation as long as the accountant does not own any interest in the corporation. a. III. and IV. only. b. III. only. c. None of the above. d. I., II., and IV. only.

B.

Before Judge Jerkins was elected to the bench, she and her law partner, Perkins, purchased a piece of property to be held in cotenancy by Perkins and Jerkins. After Jerkins was elected to the bench, Jerkins agreed to pay Perkins an annual fee to manage the property, since Jerkins's time would be severely limited by her judicial duties. Jerkins and Perkins meet every three months to discuss the status of the property. Perkins sometimes appears as an attorney in Jerkins's courtroom. Was it proper for Jerkins to make this arrangement with Perkins? a. Yes, because Jerkins acquired the property before she became a judge. b. No, because Perkins appears in cases before Jerkins's court. c. No, because judges should not engage in remunerative outside enterprises. d. Yes, if dealings concerning the property do not take up so much of Jerkins's time that her judicial duties will suffer.

B.

Building contractor Carter and his lawyer Lewis met with landowner Owens to negotiate a contract whereby Carter would construct an office building on land owned by Owens. Carter, Lewis, and Owens were the only persons present at the meeting. Ultimately the three of them worked out a written agreement, and Carter commenced work on the building. It soon became apparent that the building site required far more preparation work than Carter had contemplated when he agreed to the contract price. Carter and Owens got into a dispute about who had to pay for the additional site preparation. One important issue is whether Owens made certain oral representations to Carter during the contract negotiating session that Lewis attended. Carter contends that Owens did make the representations, and Owens contends that he did not. Lewis was present during the entire negotiating session, and she is virtually certain that Owens did not make the representations. Carter stopped work on the building and refused to proceed until Owens paid for the extra site preparation. Owens then sued Carter for specific performance of the construction contract. Carter asked Lewis to represent him as trial counsel. Lewis should: a. Agree to serve as trial counsel for Carter, because Carter is entitled to the counsel of his choice. b. Decline to serve as trial counsel for Carter, because she can foresee that she will be called as a witness. c. Agree to serve as trial counsel for Carter, because she can refuse to testify if she is called as a witness by Owens. d. Decline to serve as trial counsel for Carter, because a lawyer is not allowed to testify in a manner that is prejudicial to her client.

B.

Candidate, a member of the bar, is a candidate for judicial office in an election. Candidate personally asked several of his friends to contribute $1,000 each to kick off his campaign. After Candidate's friends made the contributions, Candidate, who was elated by the support, formed a committee to collect more contributions. Candidate then turned over the contributions to the committee and began campaigning in earnest, Is Candidate subject to discipline? a. No, unless the committee includes lawyers likely to practice before Candidate. b. Yes, because Candidate personally solicited funds. c. Yes, unless none of the original contributors was a lawyer. d. No, because Candidate turned over the funds to his committee.

B.

Criminal defendant DeVries exercised his Fifth Amendment privilege against self-incrimination and elected not to testify on his own behalf at his trial. Prosecutor Prichard presented compelling evidence of DeVries's guilt. In her closing argument to the jury, Prichard made the following statements: I. DeVries knows where he was on that fatal night. I have presented the testimony of three witnesses that he was with the victim. Did DeVries deny it? No! He sat there saying nothing. II. What are you to make of defense witness Fergus Grutz? You heard evidence that Grutz has twice been convicted of perjury. Could there be better proof that Grutz is a liar'? III. You may wonder why I cross-examined defense witness Emma Schlarp so vigorously. When you've been a prosecutor as long as I have, you can tell who is truthful and who is not. Was Emma Schlarp telling you the truth? I don't think so, do you? IV. Is DeVries guilty? That's what you have to decide, but I hope you will conclude that the evidence points only one way: guilt beyond a reasonable doubt. Which of Prichard's statements were proper? a. I., II., and IV. only. b. II. and IV. only. c. None of the above. d. I., II., and III. only.

B.

Entertainment lawyer Labrte has for many years represented country music star Spangles Tinhart. One evening, Labrte and Truhart were having a quiet business dinner together at a well-known restaurant. A brutish drunk, Duke Sirosis, lurched up to their table and in a loud voice began a vulgar and defamatory tirade against Trnhart. Everyone in the restaurant heard the vile names Sirosis called Truhart. While all of the defamatory comments about Truhart involved her personal life, about which Labrte had no real knowledge, he felt that they could not possibly be true. At Truhart's request, Labrte commenced a slander suit against Sirosis. In his answer to the complaint, Sirosis admitted making the allegedly slanderous statements, and as an affirmative defense, he alleged that the statements were entirely truthful. When the case comes to trial, would it be proper for Labrte to act as Truhart's trial counsel? a. No, unless he associates and prepares cocounsel to take over in the event he is called as a witness. b. Yes, because Labrte is not a necessary witness. c. No, because there is a possibility that Labrte may be called as a witness. d. Yes, but only if Truhart consents after full disclosure.

B.

Jillian has just been elected Judge of the Circuit Court. She has been assigned to the Probate Division. Prior to Jillian's elevation to the bench, Jillian was a partner in the law firm of Judkins & Jarvis. During her last week with the firm, Jillian filed a number of very routine, uncontested probate motions. At the time, Jillian had no idea that she would be assigned to the Probate Division. These routine probate motions have been assigned to her courtroom by a lottery system of random assignment that the Circuit Court regularly employs to assign cases. Is it proper for Jillian to rule on these motions? a. Yes, if reassignment would cause unreasonable delay. b. No, because Jillian has a conflict of interest. c. No, because judges may never role on issues where their former law firm is involved. d. Yes, because they are routine and uncontested.

B.

Judge Alpha has been assigned to try a criminal prosecution by State against Deft. Ten years prior, Alpha, while serving as a deputy attorney general in State, initiated an investigation of Deft for suspected criminal conduct. The investigation did not establish any basis for prosecution. None of the matters previously investigated is involved in or affects the present prosecution. Is it proper for Judge Alpha to try the case? a. No, if Alpha had substantial responsibility in initiating the previous investigation of Deft. b. Yes, unless Alpha might be prejudiced against Deft because of the prior investigation. c. Yes, because none of the matters previously investigated is involved in or affects the present case. d. No, if Alpha had substantial responsibility in determining that the previous investigation did not establish any basis for prosecution.

B.

Judge Alpha has recently resigned from the state trial court bench. While she was a judge and supervising activity in cases pending before Judge Beta, who was on vacation, Alpha entered an administrative order changing the courtroom in which the case of Able v. Baker was to be tried. After trial and appeal, the case was remanded for a new trial. The plaintiff in Able v. Baker has now decided to change lawyers and has asked Alpha to try the case. Will Alpha be subject to discipline if she tries this case on behalf of the plaintiff?. a. Yes, because Alpha would try the case before a judge of the court on which Alpha previously sat. b. No, because Alpha did not act as a judge with respect to a substantial matter or on the merits of the case. c. Yes, because Alpha acted officially as a judge with respect to an aspect of the case. d. No, because any information that Alpha learned about the case while acting as a judge was a matter of public record.

B.

Judge Jeffery serves on a State A trial court that has nine other judges. Her husband, Horace, is a life insurance salesman for the Amalgamated Life Insurance Company. Amalgamated is occasionally a litigant in the court on which Judge Jeffery sits. Every year Amalgamated runs a national sales contest in which the person who sells the most life insurance during the year receives a valuable prize. Horace has just learned that he is the winner this year. The prize is an all-expense-paid vacation in Europe for two people. May Judge Jeffery urge Horace to accept the prize and take her on the European vacation? a. Yes, but only if Judge Jeffery makes a public report of that portion of the prize that exceeds $150 in value. b. Yes, unless giving Horace the prize could reasonably be perceived as an attempt to influence Judge Jeffery in the performance of her judicial duties. c. No, because Amalgamated may later appear as a litigant in the court on which Judge Jeffery sits. d. Yes, because the prize was won by Horace, not by Judge Jeffery.

B.

Judge Jonathan, considered a great personal injury litigator when he was in private practice, is trying a very complicated commercial law case. He has carefully listened to the opposing attorneys' arguments and has read the briefs several times. He has found neither the oral arguments nor the briefs to be very enlightening. Sheba, a former law partner of Jonathan, is considered to be one of the leading experts on commercial law in the state. Jonathan wishes to lend the briefs to Sheba, and have her write an advisory memorandum on the issues of the case. Jonathan sincerely feels that this will enable him to render a proper judgment in a difficult case. Is it proper for Jonathan to seek such help from Sheba? a. Yes, because a judge may seek outside advice on any case. b. No, unless he gives notice to the parties and allows them time to respond to Sheba's memorandum. c. Yes, if Jonathan sincerely believes such advice is needed. d. No, unless he receives written permission from the parties prior to the consultation.

B.

Judge Junks is a loyal member of the alumni association of Heathmoor, the women's college from which she was graduated. The 25th reunion of her graduating class is coming up next June, and she has been asked to participate in some activities designed to raise money for a gift from the class to the college scholarship fund. Which of tile following activities would be proper for Judge Jones to do? I. Make a substantial personal donation to the class gift fund. II. Telephone other members of her graduating class and urge them to make a donation to the class gift fund. III. Serve on the scholarship fund committee, which devises the various fund-raising strategies. IV. Be the guest of honor at a dinner to raise funds for the class sift. V. Attend a fund-raising dinner for the class gift. a. 1., 11., IV., and V. only. b. I., III., and V. only. c. I., IV., and V. only. d. I. and V. only.

B.

Judge and Attorney were formerly law patters and during their partnership acquired several parcels of real property as co-tenants. After Judge was elected to the trial court in County, she remained a cotenant with Attorney, but left the management of the properties to Attorney. Judge's term of office will expire soon and she is opposed for reelection by two members of the bar. Attorney, who has not discussed the matter with Judge, intends to make a substantial contribution to Judge's campaign for reelection. Judge is one of fifteen judges sitting as trial court judges in County Is Attorney subject to discipline if Attorney contributes $10,000 to Judge's reelection campaign? a. No, because Attorney and Judge have a long-standing personal and business relationship. b. No, if the contribution is made to a campaign committee organized to support Judge's reelection. c. Yes, if Attorney frequently represents clients in cases tried in the trial court of County. d. Yes, because Judge and Attorney have not discussed the matter of a campaign contribution.

B.

Judge, a judge in a criminal trial court of State, wishes to serve as guardian of her father, who has been declared incompetent. Accepting the responsibilities of the position would not interfere with the performance of Judge's official duties. Although the position in all likelihood would not involve contested litigation, it would be necessary for Judge to prepare and sign various pleadings, motions, and other papers and to appear in civil court on her father's behalf. Would it be proper for Judge to undertake this guardianship? a. No, because the position will require Judge to prepare and sign pleadings, motions, and other papers. b. Yes, because the position involves a close family member and will not interfere with Judge's performance of her judicial duties. c. Yes, unless Judge receives compensation for her services as guardian. d. No, because the position will require Judge to appear in court.

B.

Judge, a state court judge, has presided over the pretrial proceedings in a case involving a novel contract question under the Uniform Commercial Code. During the pretrial proceedings, Judge has acquired considerable background knowledge of the facts and law of the matter and, therefore, is particularly well qualified to preside at the trial. Shortly before the trial date, Judge discovered that his brother owns a substantial block of stock in the defendant corporation. He determined that his brother's financial interests would be substantially affected by the outcome of the case. Although Judge believed he would be impartial, he disclosed to the parties, on the record, his brother's interest. Is it proper for Judge to hear the case? a. Yes, because Judge believes his judgment will not be affected by his brother's stockholding. b. No, unless after proper proceedings in which Judge did not participate all parties and their lawyers consent in writing that Judge may hear the case. c. Yes, because Judge is particularly well qualified to preside at the trial. d. No, because disqualification based on a relative's financial interest cannot be waived.

B.

Judge, prior to her appointment to the probate court, was a partner in Law Firm. Law Firm had an extensive probate practice. At the time of Judge's appointment, Law Firm had pending, before the court to which judge was appointed, numerous matters in which requests were being made for allowances for attorney's fees. When Judge left Law Firm, she was paid a cash settlement. She has no further financial interest in any matter handled by Law Firm. Judge is now being asked to rule on these requests for allowances for attorney's fees. Is it proper for Judge to rule on these requests? a. Yes, if these requests are not contested. b. No, because Judge was associated with Law Firm when these matters were pending. c. No, unless Judge notes on the record in each case her prior association with Law Firm. d. Yes, because Judge has no financial interest in the outcome of these cases.

B.

Justice Jacobs was on the Supreme Court of State Beta. State Beta's Supreme Court Rules provide that in capital punishment cases, any one justice of the supreme court is empowered to grant a stay of execution pending appeal to the supreme court. Justice Jacobs granted such a stay in the case of People of State Beta v. Dillon, on the ground that Dillon had been denied the effective assistance of counsel at his trial. A few months later, Justice Jacobs retired from the supreme court and went back to private law practice. In due course, the supreme court heard the appeal in the Dillon case, rejected Dillon's effective assistance of counsel contention, and affirmed the death penalty. Acting as an indigent in propria persona, Dillon then commenced a federal habeas corpus proceeding in the United States District Court for the Eastern District of State Beta, and asked that court to appoint a private lawyer to represent him in the habeas corpus proceeding. The district court appointed Jacobs to represent Dillon. A key issue in the habeas corpus proceeding is whether Dillon was deprived of the effective assistance of counsel at his trial. May Jacobs represent Dillon without getting the consent of all parties to the habeas corpus proceeding? a. No, because there is reasonable ground to doubt Jacobs's impartiality in the matter. b. No, because when Jacobs was a supreme court justice he granted a stay of execution to Dillon. c. Yes, because the respondent in the habeas corpus case is the prison warden, not the People of State Beta. d. Yes, because Jacobs was appointed by the district court, and his prior involvement in the matter is not sufficient grounds for refusing the appointment.

B.

Larry Litiger has been practicing law for two years. The following statements were broadcast in a recent radio ad: I. "Larry Litiger specializes in personal injury and divorce cases. He is a certified trial specialist." II. "Larry Litiger comes from a long line of lawyers. In fact, his father and grandfather are both judges." III. "Larry has never lost a jury trial, and 99% of his clients end up receiving some form of payment." IV. "Litiger succeeds where others fail. Don't find out the hard way, call Litiger first." Assuming that all of the above statements are tree and that Litiger complied with all the procedural requirements for running a radio advertisement, which of the statements are proper? a. I. and IV., but not II. or III. b. I. only. c. I. and II., but not III. or IV. d. II. and III., but not I. or IV.

B.

Lawyer Laden regularly represents Electratec, Inc., a manufacturer of electric kitchen appliances. One morning the president of Electratec called Laden and said excitedly: Did you read in this morning's paper about the woman who got electrocuted when she opened the door of her dishwasher? The paper said the washer was three years old, and I'm pretty sure that it was one of ours. I found our quality control records from that period, and some of our washers left the plant without proper testing. Those records should have been shredded after two years, but somehow this batch was overlooked. I'm going to send them to the shredder now, unless you tell me that I can't. Must Laden advise the president to keep the records? a. No, because at this point there is no litigation pending against the company respecting this matter. b. Yes, because the records have potential evidentiary value if the company gets sued. c. No, unless it was certain that the company was the manufacturer of the dishwasher in question. d. Yes, unless the company has a clearly established policy of shredding quality control records after two years.

B.

Lawyer Lars is defending Castco, Inc. in a suit brought in federal district court in the Second Circuit. One of the issues in the case is whether Castco violated a workplace rule promulgated by the Federal Employment Commission ("FEC"). Castco denies doing the act that allegedly violates the FEC rule. As a fall-back position, Castco argues that even if it did the act, the rule should be interpreted to exclude acts of that kind. For which of the following actions is Lars subject to discipline? I. Failing to turn over incriminating documents that Castco gave him in confidence upon his employment, and which were requested during discovery. II. Failing to cite a case directly on point decided last month by the Ninth Circuit Court of Appeals. III. Failing to notify the opposing side of a witness who can testify that the president of Castco specifically instructed one of her deputies to commit the act in question. IV. Failing to cite a three-week-old FEC decision that Lars found in a computer search and that interprets the FEC rule to include precisely the kind of act Castco allegedly committed. a. II. and IV., but not I. and III b. I. and IV., but not II. and III. c. I. and III., but not II. and IV. d. I., III, and IV., but not II.

B.

Lawyer LePage represents Prudence, the plaintiff in a sexual harassment case against defendant Dartmore Industries, Inc. Dartmore is represented by its regular corporate counsel, Clem. Prudence, who works on an assembly line, alleges that she was repeatedly harassed by the foreman on her work shift, Frank Farmer. Further, she alleges that the plant manager, Marianne Martin, was aware of Farmer's misconduct and did nothing to stop it. Prudence tells LePage that two of her co-workers on the assembly line, Will Whorley and Wendy Winston, each witnessed harassment incidents, but neither Whorley nor Winston reported the incidents to supervisory personnel. Whorley quit working for Dartmore at about the time Prudence filed her lawsuit. LePage wants to do some fact investigation before he starts discovery in the case. Which of the following best states proper conduct for LePage in interviewing the potential witnesses? a. it would be improper for LePage to interview any of these people without Clem's consent. b. LePage may freely interview Whorley and Winston, but must notify Clem that he intends to interview Farmer and Martin. c. LePage may freely interview Martin, Whorley, and Winston because they are simply third-party witnesses, but he must obtain Clem's consent to interview Farmer. d. LePage may freely interview Whorley, but must obtain Clem's consent as to Farmer, Martin, and Winston.

B.

Lawyer Lederlee was assigned by the court to defend an indigent person, former college English teacher Deniew, at her trial for the murder of her husband. The jury convicted Deniew, and she was sentenced to 40 years in prison. Lederlee's court appointment expired at the end of the trial, but he promised Deniew that he would represent her without cost in taking an appeal from her conviction. Lederlee advanced $350 on Deniew's behalf to cover the expenses of the appeal, knowing that Deniew would probably not be able to pay him back. While the appeal was pending, Deniew wrote the manuscript for a book about life in a women's prison. She hired Lederlee to negotiate a contract with a publisher to have the book published, and in return for the contract work, she promised to pay Lederlee 30% of the royalties from her book. Is Lederlee subject to discipline? a. Yes, because he advanced appeal expenses for his client, knowing that she probably could not pay him back. b. No, unless 30% of the book royalties is unreasonably high for the contract negotiation work. c. No, even if 30% of the book royalties is unreasonably high for the contract negotiation work. d. Yes, because he entered into a literary rights contract with his client while her appeal was still pending.

B.

Lawyer Liu limits her practice to tax law in State A. State A has no program for certifying specialists in any field of law. Liu has, however, been certified as a specialist in tax by the American College of Tax Counsel, a private organization that has high, rigorously enforced certification standards. Liu purchased time on local radio stations for the following advertisement: Why pay the government more than you should? I can save you at least 10% on your federal and State A income tax this year! Call Laura Liu, your hometown tax law specialist. State A has no procedure for certifying legal specialists, but I have been certified by the American College of Tax Counsel. Don't wait--call today. Laura Liu, 978-9982. Lawyer Lex believes the advertisement violates ethical standards and complains to the appropriate disciplinary authority. If the advertisement is found to be improper, it will be because: a. State A does not recognize certified specialists in any field of taw. b. The promise of 10% tax savings is likely to create unjustified expectations about the results Liu can achieve. c. Liu's advertisement is undignified and may cause listeners to lose respect for the legal profession. d. Liu's explanation of her certification is insufficient.

B.

Lawyer Lockwood represents defendant Downs in a drug smuggling case. Downs is in pretrial custody in a distant city and cannot be reached by telephone. One key issue in Downs's case is on the cutting edge of search and seizure law, and Lockwood believes that he needs help to deal with the issue competently. Lockwood's former law professor is a nationally known expert on search and seizure law. Lockwood calls his professor to ask lot his help, and also asks that the professor keep this information confidential. To frame the issue accurately, Lockwood tells the law professor some information that Downs revealed to Lockwood in confidence. Lockwood does not tell the professor tile name of his client. Is Lockwood subject to discipline r0r disclosing Downs's confidential information to the professor? a. Yes, unless Downs had specifically authorized Lockwood to make such a disclosure. b. No, because the disclosure was necessary to effectively carry out the representation. c. No, because Lockwood did not reveal his client's name. d. Yes, if the professor was not licensed to practice in that jurisdiction.

B.

Lawyer London and her nonlawyer friend Ferguson created a partnership to serve people who want to invest in commercial real estate. Ferguson, a licensed real estate developer, finds promising commercial real estate projects, brings together groups of investors, and works with local planning authorities to gain approval for the projects. London drafts the legal documents for the projects, assists the investors with the legal technicalities, advises the investors on their tax liabilities, and does whatever legal work the investors need in connection with management and operation of the projects. London and Ferguson charge the investors a single fee for their work, and they divide the partnership profits 50%-50%. Is London subject to discipline? a. No, provided the investors consent after full disclosure of the potential conflicts of interest. b. Yes, because Ferguson and London are partners in the business. c. No, because Ferguson does only development work, and London does only legal work. d. Yes, because she is aiding Ferguson in the unauthorized practice of law.

B.

Lawyer Loomis is the head of the in-house law department of Darlington KiddieWear Corp., which has its principal place of business in State A. Under the law of State A, it is a felony to manufacture or sell children's sleepwear that is not fire retardant. The president of Darlington informed Loomis in confidence that Darlington is stuck with a whole warehouse full of children's pajama fabric that does not meet State A's fire standards, and that to avoid financial disaster Darlington will use the fabric to make children's sleepwear and take its chances on legal liability. Loomis was unable to convince the president to change his mind; she then raised the issue with Darlington's board of directors, which ratified the president's decision. Will Loomis be subject to discipline if she resigns as house counsel and reports the matter to the appropriate State A law enforcement authorities? a. Yes, because there is no adequate reason for permissive withdrawal on these facts. b. No, because she is entitled to reveal this type of confidential information. c. Yes, because she is required to preserve the corporation's confidential information even after she resigns. d. No, because her duty to preserve confidential information ceases with her resignation as house counsel.

B.

Lawyers Abner, Baker and Clark formed a law partnership; each contributed $100,000 in capital to get the firm started. Their partnership agreement provided that when a partner dies, the firm will make certain payments to the dead partner's named beneficiary. Sometime later, Clark died, leaving his daughter Clara, a doctor, as his sole beneficiary. Under the partnership agreement, the firm plans to make the following payments to Clara: $100,000, which represents Clark's share of the firm's assets, as measured by his capital contribution; $45,000, which represents Clark's share of fees that had been earned but not collected from clients at Clark's death; and a $125,000 death benefit, representing a percentage of Clark's earnings the year prior to his death and payable in monthly installments. Which of the following payments may the firm make to Clara? a. $100,000 for Clark's share of the firm's assets. b. $270,000, which includes Clark's share of the firm's assets, Clark's share of uncollected fees, and the death benefit. c. $145,000 for Clark's share of the firm's assets and Clark's share of uncollected fees. d. $170,000, which represents the death benefit and Clark's share of uncollected fees.

B.

Paul Poller brought a civil action to recover damages for personal injuries he suffered as the victim of alleged police brutality inflicted by defendant police officers Able, Baker, and Carter. The trial was widely reported by the media. The jury returned a verdict in favor of Poller and against the three police officers for $500 million. When Trial Judge Johnston received the verdict, he was shocked by the size of the award. Before dismissing the jurors, Judge Johnston directed the following statements to the jury: When you people were sworn in as jurors in this case, you promised that you would deliver a verdict based on the evidence, and that you would not be swayed by passion or prejudice. You have failed in those duties and made a mockery of justice. You should be ashamed of yourselves. He then dismissed the jury, and the defense lawyers renewed their motion for judgment as a matter of law and, alternatively, moved for a new trial. Judge Johnston announced that he would rule on the motions the following Monday at 10 a.m. in open court. The press reports of the verdict and the judge's comments to the jury created a great public tumult in the city where the case was tried. On the following Monday, the courtroom wag jammed with reporters. Primarily for the purpose of educating the reporters, Judge Johnston first gave a detailed explanation of the legal requirements for granting a renewed motion for judgment as a matter of law and for granting a new trial motion. He then granted the renewed motion for judgment as a matter of law and, alternatively, the motion for a new trial. Were Judge Johnston' s actions proper? a. The statements to the jury were proper, but the communication with the reporters was not. b. The communication with the reporters was proper, but the statements to the jury were not. c. Both the statements to the jury and the communication with the reporters were proper. d. Neither the communication with the reporters nor the statements to the jury were proper.

B.

Pilate, a local assistant district attorney, has just finished prosecuting a case against Dudley Defendant, who was accused of committing a serious felony. Pilate believed he had a strong case, but Justinian, the judge trying the case, ruled to acquit the defendant. Justinian is running for reelection in a contested campaign. The judicial election will occur in two months. Immediately after Dudley's trial is over, Pilate makes the following statement to reporters assembled on the courthouse steps: Dudley Defendant is walking away front this courthouse a free man only because of the erroneous rulings of Judge Justinian. In the many years that I have tried cases for the state in this courthouse, I have become aware that in every case heard by Judge Justinian where I have appeared before him as prosecutor, there have always been clearly erroneous rulings in favor of the defendant. I am going to do everything in my power to see that Justinian is not reelected, and I am going to work very hard for the election of Genseric, his opponent in the forthcoming judicial election. Pilate has personally tried four cases in front of Justinian. Is Pilate's statement proper? a. Yes, because Pilate spoke out after the verdict had been rendered. b. No, because Pilate has brought the administration of justice into disrepute. c. Yes, because the public needs to be in-formed about incompetent judges. d. No, because Pilate is a public official and he should not have criticized another public official.

B.

Plaintiff and Defendant are next-door neighbors and bitter personal enemies. Plaintiff is suing Defendant over an alleged trespass. Each party believes, in good faith, in the correctness of his position. Plaintiff is represented by Attorney Alpha, and Defendant is represented by Attorney Beta. After Plaintiff had retained Alpha, he told Alpha "I do not want you to grant any delays or courtesies to Defendant or his lawyer. I want you to insist on every technicality." Alpha has served Beta with a demand to answer written interrogatories. Beta, because of the illness of his secretary, has asked Alpha for a five-day extension of time within which to answer them. Is Alpha subject to discipline if she grants Beta's request for a five-day extension? a. No, because Beta was not at fault in causing the delay. b. No, unless granting the extension would prejudice Plaintiff's rights. c. Yes, unless Alpha first informs Plaintiff of the request and obtains Plaintiff's consent to grant it. d. Yes, because Alpha is acting contrary to her client's instructions.

B.

Preacher is a law school graduate but is not a licensed member of the bar. After Preacher graduated from law school, he felt called to the ministry, received a degree in divinity, and was formally ordained as a minister of his faith. Preacher is now pastor of a local church, where Alfred Attorney is a member of the congregation. Preacher has been very disturbed about the high rates of divorce and the breakdown in American family life. Therefore, he holds frequent "family counseling sessions" where, among other things, he explains to the parishioners who attend these sessions many of the legal ramifications of divorce, alimony, child support, and child custody. These sessions are usually followed by question-and-answer periods, during which Preacher gives legal advice to parishioners who cannot afford a lawyer. Preacher knows that the legislature has passed a new marriage dissolution law that changes the law substantially from what Preacher was taught in law school. Preacher asks Alfred if he will prepare an outline and a memorandum fully explaining the new law, so that Preacher will be better informed for the sessions with his parishioners. If Alfred agrees to do this, is he subject to discipline'? a. No, since Alfred has a duty to help educate the public regarding the law. b. Yes, since Alfred is assisting in the unauthorized practice of law. c. Yes, since Preacher may bring Alfred before the State Bar Disciplinary Committee if Alfred makes any mistake of law in the memorandum. d. No, if marriage counseling is an important part of Preacher's duties as pastor

B.

Pros, an elected prosecutor in City, plans to run for reelection in six months. Last year two teenage girls were kidnapped from a shopping center and sexually assaulted. The community was in an uproar about the crime and put pressure on Pros to indict and convict the assailant. Four months ago, Deft was arrested and charged with the crimes. The trial is scheduled to begin next week. Pros met with the police chief last week to review the evidence in the case. At that time, Pros first learned that, before they were interviewed by the detective in charge of sexual assault crimes, the two victims had been tape-recorded discussing the case between themselves in an interview room. Reviewing the tape, Pros realized that the girls' descriptions of the assailant differed significantly in terms of height, weight, and hair color. When officially interviewed, however, their descriptions matched almost perfectly. Deft's appointed counsel was busy handling large caseload of indigent defendants and neglected to seek access to the prosecution's investigative file. Pros was virtually certain that Deft's counsel was unaware of the tape recording. Given the other evidence in the case, Pros reasonably believed that the girls accurately identified Deft as their assailant. Pros did not reveal the existence of the tape to defense counsel. Is Pros subject to discipline? a. No, unless Deft's counsel submitted a request for all mitigating or exculpatory evidence before the start of trial. b. Yes, because the tape raises a legitimate question about the victims' eyewitness identification of Deft as the assailant. c. No, because under the adversary system of criminal justice, it is expected that each party will marshal the evidence best supporting its own position. d. Yes, unless Pros reasonably believed that the girls accurately identified Deft as their assailant.

B.

Shortly after the county grand jury handed down an indictment for armed robbery against a fugitive, the district attorney met with the working press outside the door to the grand jury room. He tells the reporters that he is limited as to what he can say about pending cases and will make only three statements. Any other questions will be answered with "no comment." Which of the following statements would not be proper to make to the press? A) "The fugitive has been indicted by the grand jury for armed robbery, but like all other American citizens he should be considered innocent until proven guilty." B) "He was indicted after grand jury testimony by two credible witnesses." C) "The public should be warned that this man is a fugitive and is considered to be armed and dangerous." D) "No comment."

B.

Solo practitioner Proctor is one of only three lawyers in the small town of Sandy Gulch. Proctor is presently defending client Cridley in a criminal action for assault and battery. This morning one of Proctor's regular clients, the Sandy Gulch Gas & Grocery, asked Proctor to sue Cridley to recover $638.64 that is past due on Cridley's gasoline and grocery charge account. Would it be proper for Proctor to represent the Sandy Gulch Gas & Grocery in the charge account case? a. No, unless the other two lawyers in town are disqualified from representing the Sandy Gulch Gas & Grocery. b. No, unless both Cridley and the Sandy Gulch Gas & Grocery consent after full disclosure. c. Yes, unless there is a substantial relationship between the charge account case and the assault and battery case. d. Yes, unless Proctor has gotten confidential information from Cridley that would be relevant to the charge account case.

B.

State Alpha lawyer LaFrank represents Inventex Corp., a State Alpha corporation that owns a valuable United States patent. State Beta lawyer Levin represents Demonics, Inc., a State Beta corporation. Inventex believes that Demonics is infringing tile Inventex patent, and for the past three months LaFrank has been negotiating with Levin, seeking an amicable resolution of the dispute. The negotiations have broken down, and Inventex has decided to sue Demonics for infringement. Venue in an action for patent infringement is proper in any United States judicial district where the defendant or its agent resides or can constitutionally be served with process. For tactical reasons, LaFrank wants venue in State Alpha. She believes, but is not certain, that Demonics has a branch sales office somewhere in State Alpha. To find out for sure, LaFrank telephones Demonics's headquarters in State Beta and asks to speak with the vice president for sales. Without identifying herself, she asks the vice president where in State Alpha she can find a sales office for Demonics's products. The vice president politely tells her the address of the office and the name of Demonics's head sales agent in State Alpha. Is LaFrank subject to discipline? a. Yes, because when she talked with the vice president, she did not identify herself as counsel for Inventex. b. Yes, because she talked with the vice president about the matter without getting Levin's consent. c. No, because at the time she talked with the vice president, Inventex had not yet sued Demonics. d. No, because her conversation with the vice president did not concern the substance of the dispute between the parties.

B.

Deft was on trial for the murder of Victim, who was killed during a bar brawl. In the course of closing arguments to the jury, Prosecutor said, "Deft's entire defense is based on the testimony of Wit, who said that Victim attacked Deft with a knife before Deft struck him. No other witness testified to such an attack by Victim. I don't believe Wit was telling the truth, and I don't think you believe him either." Was Prosecutor's statement proper? a. No, because Prosecutor alluded to the beliefs of the jurors. b. Yes, if Prosecutor accurately stated the testimony in the case. c. No, because Prosecutor asserted his personal opinion about Witness's credibility. d. Yes, if Prosecutor, in fact, believed Wit was lying.

C.

The Department of Children's Services ("DCS"), a state agency, has removed Clara's children from her home and is attempting to terminate her parental rights. Clara retains lawyer Laura to fight DCS's actions. Prior to the first hearing on the matter, a lawyer from DCS contacted Ben, Clara's brother, to set up an interview. The lawyer told Ben that he wanted to talk to him about Clara, and that the interview would last for about one hour. Ben, who is childless, had never had any contact with DCS before, and did not know what his duties were. He told the lawyer he would get back to him. Ben then called Laura. He told Laura that he does not want to talk to DCS because he is afraid he might inadvertently say something that will hurt Clara's case. Ben asked Laura how he should proceed. Laura replied, "My advice to you is to simply refuse the interview. You do not have to talk to them, and your sister will be better off if you don't. If they really want to talk to you, they can get a subpoena." Is Laura subject to discipline? a. No, but only if she also advised Ben to seek the advice of independent counsel. b. No, because Ben is Clara's brother and he did not want to hurt her case. c. Yes, because she is attempting to secure the noncooperation of a witness. d. Yes, because she gave advice to an un-represented party.

B.

The judicial district in which Judge sits has a rule that allows litigants two postponements as a matter of right. After that, a litigant who moves for a postponement must convince the presiding judge that a postponement is appropriate. Judge routinely grants additional postponements because, in her view, "What harm is done if one of the litigants wants a postponement? The worst that can happen is that the parties have more time to negotiate and thus are more likely to settle." Are Judge's actions proper? a. Yes, because Judge is exercising her judicial discretion. b. No, because Judge should expedite the determination of matters before her. c. No, because judges have no official obligation to encourage private settlements. d. Yes, because a party objecting to a postponement can seek appellate review.

B.

Union Bank and Trust Company maintains a list of Approved Estate and Trust Lawyers as a service to Union customers who seek Union's advice on estate planning matters. When lawyer Lieu opened her trust and estate practice in town, she asked other lawyers how to get on Union's approved list. They explained that Union lists lawyers who regularly name Union in wills and trust agreements they draft for clients who need an institutional executor or trustee. Union is one of the most stable and reputable banks in the state, and its fees for executor and trustee services are competitive with those of similar institutions. In light of what she has been told by the other lawyers, may Lieu seek to have her name included on Union's list? a. Yes, because those who use Union's list are already Union customers. b. No, because a tacit condition of being on the list is regularly to name Union as executor or trustee. c. No, because a lawyer must not solicit business through an intermediary. d. Yes, because naming Union causes no harm to clients who need an institutional executor or trustee.

B.

Walters is insured under an auto liability policy issued by Farmers Insurance Company. The policy requires Farmers to provide a lawyer to defend Walters, and it requires Walters to cooperate in the defense. Walters had an accident and was sued. In a sworn statement to Farmers' insurance investigator, Walters told a story that showed he was clearly not at fault. Based on that story, Farmers rejected plaintiff's offer to settle the case for a modest sum. Farmers hired attorney Chen to represent Walters at the trial of the case. Shortly before trial, Walters told Chen in confidence that he had lied to the investigator, and he recounted facts that showed he was clearly at fault in the accident. Chen realized that under the applicable state law, Walters' falsehood was a breach of the "cooperate in the defense" clause, and that it relieved Farmers of any further duties to Walters. At this juncture, what should Chen do? a. Promptly advise Walters of the legal consequences of his false statement, and continue representing Walters and Farmers in the matter as best he is able in the circumstances. b. Promptly seek the court's permission to withdraw from the matter, without revealing Walters' confidential statement to anyone. c. Promptly advise Farmers of the situation and carry out Farmers' instructions as to how to dispose of the matter. d. Promptly advise Walters that his best interests will be served by reverting to the story he told originally to the insurance investigator.

B.

While presiding over the trial of a highly publicized antitrust case, ABCO v. DEFO, Judge received in the mail a lengthy letter from Attorney, a local lawyer. The letter discussed the law applicable to ABCO v. DEFO. Judge knew that Attorney did not represent either party. Judge read the letter and, without mentioning its receipt to the lawyers in the pending case, filed the letter in his general file on antitrust litigation. Later, after reading the trial briefs in ABCO v. DEFO, Judge concluded that Attorney's letter better explained the law applicable to the case pending before him than either of the trial briefs. Judge followed Attorney's reasoning in formulating his decision. Was it proper for Judge to consider Attorney's letter? a. No, because Attorney is not of record as counsel in the case. b. No, unless Judge, prior to rendering his decision, communicated its contents to ail counsel and gave them an opportunity to respond. c. Yes, if Attorney did not represent any client whose interests could be affected by the outcome. d. Yes, because Judge did not initiate the communication with Attorney.

B.

Wilma is the only living child of widower Warner, age 83. Wamer's main asset is a 51% partnership interest in Mobiland, Ltd., a wealthy real estate syndicate that owns and operates mobile home parks throughout the state. Wilma is married to attorney Atwater. One of Atwater's regular clients, Christopher, asks Atwater to represent him in negotiating the sale of 3,000 acres of roadside property to Mobiland. Mobiland is represented by its own lawyer in the matter. May Atwater represent Christopher? a. Yes, because Atwater has no significant personal interest in Mobiland. b. Yes, but only if Christopher consents after full disclosure of Atwater's connection with Warner. c. No, even if Christopher consents after full disclosure of Atwater's connection with Warner. d. No, because to do so would create an appearance of impropriety.

B.

Acton, a certified public accountant, has proposed to Attorney, a recognized specialist in the field of tax law, that Acton and Attorney form a partnership for the purpose of providing clients with tax-related legal and accounting services. Both Acton and Attorney have deserved reputations of being competent, honest, and trustworthy. Acton further proposes that the announcement of the proposed partnership, the firm stationery, and all public directory listings clearly state that Acton is a certified public accountant and that Attorney is a lawyer. Is Attorney subject to discipline if he enters into the proposed partnership with Acton? a. No, because the partnership will assure to the public high-quality services in the fields of tax law and accounting. b. Yes, because Attorney would be receiving fees paid for other than legal services. c. Yes, because one of the activities of the partnership would be providing legal services to clients. d. No, if Attorney is the only person in the partnership who gives advice on legal matters.

C.

Alpha & Beta, a general partnership, is a litigation firm practicing in State, it hires new law school graduates as associates. These new lawyers are largely left to their own resources to practice law. The law firm accepts many small litigation matters and assigns them to the associates for training purposes. No senior partners are assigned to supervise this work. It is assumed that if an associate needs help on a case, he or she will seek the guidance of a more senior attorney. Client retained Alpha & Beta to pursue a claim for breach of contract against City. Associate, a first year associate, was assigned Client's case. Associate failed to comply with the applicable 30-day notice requirement for filing a complaint against City, and Client lost the chance to recover $5,000 owed to Client by City. When the complaint was dismissed for failure to comply with the notice requirement, Associate instead told Client that the case was dismissed on the merits. Which of the following statements are correct? I. The law firm of Alpha 8: Beta is subject to discipline for failure to supervise Associate. II. The individual partners of Alpha & Beta are subject to discipline for failure to make reasonable efforts to establish a system providing reasonable assurance that all lawyers in the firm comply with the roles of professional conduct. III. Associate, an unsupervised subordinate lawyer, is subject to discipline for making misrepresentations to Client. IV. Both the law firm of Alpha & Beta and Associate are subject to civil liability for Client's loss. a. I, III, and IV, but not II b. II and IV, but not I or III c. II, III, and IV, but not I d. I, II, III, and IV

C.

Arlington is an attorney engaged in private practice in the city of New Novgorod. Arlington has many friends who belong to the Ancient Society of Sultans, a fraternal and charitable organization with chapters throughout the state. Arlington is not a member of the Society, but knows a number of its officers socially and has performed legal work for them on matters unrelated to the Society. The officers of the Society are sometimes consulted by members who have legal problems. The officers, being very pleased with the quality of Arlington's work, often refer such members to Arlington. Arlington has never asked the officers for such referrals, but is, of course, very pleased since he has earned substantial fees from these referrals. The Society is presently organized as an unincorporated association, but the leaders are interested in incorporating the Society under the State Nonprofit Corporation Act. One of the officers asks Arlington what his fee would be for incorporating the Society. Arlington tells the officer that he is very grateful for the client referrals from the Society and, as a token of his appreciation, he will not charge a fee for the incorporation work. Is Arlington subject to discipline? a. No, because Arlington did not solicit the referrals. b. No, because an attorney always has the option of waiving a fee. c. Yes, because an attorney must not give something of value in return for client referrals. d. Yes, since only clients unable to pay should be given free legal services.

C.

Assistant District Attorney Krytzer has been assigned to prosecute Abdala for petty theft and attempted sale of stolen property. Abdala's arrest was the result of information provided by O'Leary, a local pawnbroker. O'Leary has himself been in trouble with the law on prior occasions, but he is not suspected of any present crime. Abdala is represented by Public Defender Fuchinello. Krytzer wants to interview O'Leary for possible use as a prosecution witness. Which of the following is most nearly correct? a. Krytzer may interview O'Leary without Fuchinello's consent, but he would be subject to discipline for inquiring about O'Leary's prior criminal record. b. Krytzer may interview O'Leary, but only with Fuchinello's consent, and Krytzer would be subject to discipline for inquiring about O'Leary's prior criminal record. c. Krytzer may interview O'Leary without Fuchinello's consent. d. Krytzer would be subject to discipline if he interviewed O'Leary without Fuchinello's consent.

C.

Attorney Ace placed an advertisement that ran daily in the classified section of the Pikeville News Journal, a newspaper of general circulation, widely read in the Pikeville area where Ace practiced. Besides stating Ace's office address and telephone number, and identifying Ace as a licensed attorney, the ad included the following statement: DIVORCES - LOW RATES!!! Just $l00, plus costs for uncontested divorces. According to bar association surveys, the "low average" fee in the Pikeville area for an uncontested divorce is $125, plus costs. Is Ace's advertising proper? a. Yes, because the legal profession imposes no substantive limitations on comparative advertising. b. No, because the ad is in bad taste and constitutes a self-serving attempt to solicit business at the expense of fellow attorneys. c. Yes, because Ace's rates really are low. d. No, because Ace fails to state his range of fees for contested divorces.

C.

Attorney Alpha is a candidate in a contested partisan election to fill a vacancy on the district court bench. Attorney Beta has known Alpha since law school days, and the two attorneys often appear in the same court. Beta thinks that Alpha is an intelligent and competent attorney. However, Beta also believes that Alpha has always been "an arrogant hothead," and that a person with that kind of personality lacks the temperament to serve on the bench, where Alpha's tendencies toward arrogance might grow even worse. Newsmann, a reporter for The Daily News, a local general circulation newspaper, stood outside the courthouse one afternoon asking attorneys for their opinions of various candidates for the bench. When Beta was questioned, he told the reporter, "I'm supporting attorney Gamma for the district court vacancy, because I think that Gamma's opponent, Alpha, lacks the proper judicial temperament." Beta's comments were published the next day in The Daily News. Was Beta's conduct proper? a. No, because his comments might tend to bring the judiciary into disrepute. b. No, because attorneys should not comment on the qualifications of judicial candidates. c. Yes, because Beta honestly believes that Alpha lacks judicial temperament. d. Yes, because Beta is not a candidate for judicial office.

C.

Attorney Aoki represents client Carson, the plaintiff in a personal injury suit arising out of a tour bus accident in Hawaii. Nearly all of the eyewitnesses were tourists who have now returned home to the mainland. Without notifying the defense attorney, Aoki has interviewed most of the witnesses by phone. By far the most compelling witness, and the one most favorable to Carson, is Willa. Willa is a librarian who lives in North Dakota and had spent the bulk of her life savings on a vacation to Hawaii It wax on this vacation that Willa witnessed the tour bus accident. Willa is a very appealing witness, and Aoki is confident that if a jury saw her testify personally, Carson would win his suit. Aoki tells Willa that if she is willing to come to Hawaii for one week to testify, he will pay for plane tickets, an oceanfront hotel room at a first class hotel, all meals, and one week's salary for her lost time. This is the same offer Aoki makes to all witnesses traveling to testify in any of his cases. Willa, who cannot believe her good fortune, readily agrees. Is Aoki subject to discipline? a. Yes, because a lawyer may not offer an inducement to a witness to testify. b. No, unless the trip and accommodations are substantially more expensive than Willa could afford. c. No, because a lawyer may pay a witness's reasonable expenses and lost wages. d. Yes, because he interviewed the witnesses without notifying the defense attorney.

C.

Attorney Archer is representing Davis in the civil case of Preston v. Davis, which arose out of a business deal gone sour. One evening after court was out of session, but with the trial set to resume the next day, Archer attended a $5,000 per person charity fundraising dinner. When he found his assigned seat, he was shocked to find he was seated next to Preston, the plaintiff in the suit. Archer asked the hostess if she could change his seat, but she said it would be impossible. Determined to make the best of it, Archer and Preston did not discuss the case but made small talk about the charity, the weather, etc. They soon found they had much in common, including a love of sports. After a long, enjoyable evening of discussing their favorite teams, Preston gets up to leave. Preston turns to Archer and says, "You're not a bad fellow after all. It has been a real pleasure talking to you this evening. First thing in the morning, I'm going to talk to my lawyer 'about reaching an agreeable settlement in this case. I'll have her call you." The next day, Preston calls lawyer Layton and tells her, "After talking to Archer last night at tile fundraiser, I have decided to settle the case for the amount proposed in our last negotiating session. If Davis agrees to it, please notify the court and draw up the appropriate papers." Furious, Layton does as Preston asks, but reports Archer to the proper disciplinary authorities. Is Archer subject to discipline ? a. No, because Archer did not know he would be seated next to Preston and asked to have the seating assignment changed as soon as he discovered it. b. Yes, because this is an improper ex parte communication while the matter is still pending. c. No, because they discussed the charity, the weather, and sports. d. Yes, because he communicated with a represented party without his attorney's consent.

C.

Attorney agreed to represent Able, a client, in bringing a lawsuit. Attorney and Able executed Attorney's preprinted retainer form that provides, in part: "The client agrees to pay promptly Attorney's fees for services, in addition, the client and Attorney agree to release each other from any and all liability arising from the representation. The client agrees that Attorney need not-return the client's file prior to receiving the client's executed release. Attorney agrees to return the client's file promptly upon receipt of all fees owed and of the client's executed release." During their initial meeting, Attorney recommended that Able consult independent counsel before signing the retainer agreement, but Able chose not to do so. Attorney reasonably believes that his fee is fair and that the quality of his work will be competent. Is Attorney's retainer agreement with Able proper? a. Yes, because Attorney furnished consideration by agreeing to release Able from liability and to return Able's files. b. Yes, because Attorney reasonably believes that his fee is fair and that the quality of his work will be competent. c. No, because Attorney is attempting to limit prospectively his liability for malpractice. d. No, because Attorney uses a preprinted form for all retainers.

C.

Attorney has been representing Client in a matter in litigation. During protracted pretrial proceedings, Client complained bitterly about the time and expense involved and insisted that Attorney take steps to terminate the pretrial proceedings. Attorney believes that to do so would jeopardize Client's interests and has so informed Client. Attorney believes that the case cannot be adequately prepared for trial without further pretrial proceedings that will require an additional six months' delay and involve further expense. Client insists that Attorney forego any further pretrial proceedings and set the case for trial at the earliest available date. There are several other competent lawyers who are willing to undertake the representation. Is it proper for Attorney to ask leave of the court to withdraw.'? a. No, because Attorney must follow Client's instructions. b. No, unless Client consents to Attorney's withdrawal. c. Yes, because Client's conduct makes it unreasonably difficult for Attorney to represent Client effectively and competently. d. Yes, because a lawyer may discontinue representation in a civil case at any time before trial.

C.

Attorney in his capacity as part-time .assist .ant county attorney represented County m a criminal non-support proceeding against Husband. This proceeding concluded with an order-directing Husband to pay or be jailed. Husband refused to pay. Attorney, pursuant to applicable rules, is permitted to maintain a private law practice. Wife has discovered some assets of Husband. Attorney now has accepted employment from Wife to maintain a civil action against Husband to recover out of those assets arrearages due to Wife under Wife's support decree. Attorney did not obtain consent from the county attorney or from Husband to represent Wife in the civil action. Is Attorney subject to discipline for accepting employment in Wife's civil action against Husband? a. No, because Attorney's responsibility in his public employment has terminated. b. No, because Attorney is representing Wife's interest in both the criminal and the civil proceedings. c. Yes, because Attorney had personal and substantial responsibility in the first proceeding. d. Yes, because Attorney did not obtain Husband's consent to the representation.

C.

During a brief recess in jury deliberations in a criminal case, juror Jimmerson telephoned a friend of hers, attorney Aulet, and asked for help in understanding a legal concept that was puzzling the jury--the meaning of "beyond a reasonable doubt." Aulet explained the term as best he could, given the circumstances and shortness of time. Is Aulet subject to discipline? a. No, unless he was not competent in the field of criminal law. b. Yes, because given the circumstances and shortness of time, his explanation may have been misleading. c. No, unless he was in some manner connected with the case. d. Yes, because he communicated with a juror about a pending case.

D.

Attorney is a member of the bar and a salaried employee of the trust department of Bank. As part of his duties, he prepares a monthly newsletter concerning wills, trusts, estates, and taxes, which Bank sends to all of its customers. The newsletter contains a recommendation to the customer to review his or her will in light of the information contained and, if the customer has any questions, to bring the will to Bank's trust department where the trust officer will answer any questions without charge. The trust officer is not a lawyer. If the trust officer is unable to answer the customer's questions, the trust officer refers the customer to Attorney. Is Attorney subject to discipline for the foregoing? a. Yes, because Attorney is giving legal advice to persons who are not his clients. b. No, because Attorney is a member of the bar. c. Yes, because Attorney is aiding Bank in the unauthorized practice of law. d. No, because no charge is made for Attorney's advice.

C.

Attorney placed Associate, recently admitted to the bar, in complete charge of the work of the paralegals in Attorney's office. That work consisted of searching titles to real property, an area in which Associate had no familiarity. Attorney instructed Associate to review the searches prepared by the paralegals, and thereafter to sign Attorney's name to the required certifications of title if Associate was satisfied that the search accurately reflected the condition of the title. This arrangement enabled Attorney to lower office operating expenses. Attorney told Associate that Associate should resolve any legal questions that might arise and not to bother Attorney because Attorney was too busy handling major litigation. Is it proper for Attorney to assign Associate this responsibility? a. Yes, if the paralegals are experienced in searching titles. b. Yes, because Attorney is ultimately liable for the accuracy of the title searches. c. No, because. Attorney is not adequately supervising the work of Associate. d. No, unless it enables Attorney to charge lower fees for title certification.

C.

Attorney represented Buyer in a real estate transaction. Due to Attorney's negligence in drafting the purchase agreement, Buyer was required to pay for a survey that should have been paid by Seller, the other party to the transaction. Attorney fully disclosed this negligence to Buyer, and Buyer suggested that he would be satisfied if Attorney simply reimbursed Buyer for the entire cost of the survey. Although Buyer might have recovered additional damages if a malpractice action were filed, Attorney reasonably believed that the proposed settlement was fair to Buyer. Accordingly, in order to forestall a malpractice action, Attorney readily agreed to make the reimbursement. Attorney drafted a settlement agreement, and it was executed by both Attorney and Buyer. Was Attorney's conduct proper? a. No, because Attorney settled a ease involving liability for malpractice while the matter was still ongoing. b. Yes, because Attorney reasonably believed that the proposed settlement was fair to Buyer. c. Yes, if Attorney advised Buyer in writing that Buyer should seek independent representation before deciding to enter into the settlement agreement. d. No, unless Buyer was separately represented in negotiating and finalizing the settlement agreement.

C.

Attorney represented Plaint, who sued Deft for injuries Plaint sustained in a car accident. Prior to trial, Attorney interviewed Wit, who stated that she had observed Deft drinking heavily hours before the accident. Unfortunately, on the eve of trial, Wit informed Attorney that Wit was ill and could not testify at trial. Attorney tried but could not obtain a continuance. As a result, Plaint's direct case rested solely on Plaint's testimony that Deft was speeding and that Deft's car crossed the center line and hit Plaint's car. Deft testified that he was driving safely in compliance with all rules and that the accident was entirely Plaint's fault. On cross examination, Attorney asked Deft, "Isn't it a fact that you were drinking prior to the accident?" Deft answered that he had not consumed alcoholic beverages on the day of the accident. In summation to the jury, Attorney stated: "Ladies and gentlemen of the jury, you and I know that Deft lied when he stated that he had not consumed alcoholic beverages on the day of the accident. We know that he was impaired." On which of the following grounds, if any, is Attorney subject to discipline? I. Attorney's question to Deft implying that Deft had consumed alcoholic beverages when Attorney knew that he could not offer evidence of Deft's drinking. II. Attorney's statement to the jury asserting that Attorney knew that Deft was drank when no evidence in the record supported this allegation. III. Attorney's statement asserting a personal belief that Deft was drunk and lying. I, Il, and III Neither I, II, nor III II and III, but not I I and II, but not III

C.

Attorney represented Seller in negotiating the sale of his ice cream parlor. Seller told Attorney in confidence that, although the business was once very profitable, recent profits have been stable but modest. As the negotiations proceeded, Buyer appeared to be losing interest in the deal. Hoping to restore Buyer's interest, Attorney stated, "The ice cream business is every American's dream: happy kids, steady profits, and a clear conscience." Buyer bought the ice cream parlor but was disappointed when his own profits proved to be modest. Is Attorney subject to discipline? a. Yes, because Attorney exaggerated the profitability of the business. b. No, because Attorney represented Seller, not Buyer. c. No, because Attorney's statement constitutes acceptable puffing in negotiations. d. Yes, because Attorney made a false statement of fact to Buyer.

C.

Attorney represents Client, a famous politician in an action against Newspaper for libel. The case has attracted substantial publicity, and a jury trial has been demanded. After one of the pretrial hearings, as Attorney left the courthouse, news reporters interviewed Attorney. In responding to questions, Attorney truthfully stated: "The judge has upheld our right to subpoena the reporter involved (identified in our motion as Jane Doe) and question her on her mental impressions when she prepared the article." Is Attorney subject to discipline for making this statement? a. No, because the trial has not commenced. b. Yes, because Attorney identified a prospective witness in the case. c. No, because the statement relates to a matter of public record. d. Yes because prospective jurors might learn of Attorney's remarks.

C.

Four years ago, Alpha was a judge in a state court of general jurisdiction and heard the civil case of Plaintiff against Defendant in which Plaintiff prevailed and secured a judgment for $50,000 that was sustained on appeal. Since then Alpha has resigned from the bench and returned to private practice. Defendant has filed suit to enjoin enforcement of the judgment on the grounds of extrinsic fraud in its procurement. Plaintiff has now asked Alpha to represent Plaintiff in defending the suit to enjoin enforcement. Is it proper for Alpha to accept the representation of Plaintiff in this matter? a. Yes, if Alpha's conduct of the first trial will not be in issue. b. No, unless Alpha believes the present suit is brought in bad faith. c. No, because Alpha had acted in a judicial capacity on the merits of the original case. d. Yes, because Alpha would be upholding the decision of the court.

C.

Attorney, who had represented Testator for many years, prepared Testator's will and acted as one of the two subscribing witnesses to its execution. The will gave 10% of Testator's estate to Testator's housekeeper, 10% to Testator's son and sole heir, Son, and the residue to charity. Upon Testator's death one year later, Executor, the executor named in the will, asked Attorney to represent him in probating the will and administering the estate. At that time Executor informed Attorney that Son had notified him that he would contest the probate of the will on the grounds that Testator lacked the required mental capacity at the time the will was executed. Attorney believes that Testator was fully competent at all times and will so testify, if called as a witness. The other subscribing witness to Testator's will predeceased Testator. Is it proper for Attorney to represent Executor in the probate of the will? a. Yes, because Attorney is the sole surviving witness to the execution of the will. b. No, because Attorney will be representing an interest adverse to Testator's heir at law. c. No, because Attorney will be called to testify on a contested issue of fact. d. Yes, because Attorney's testimony will support the validity of the will.

C.

Booker is employed as an accountant for Hyrax Corporation. Several months ago, Hyrax's president, Potsdam, noticed that there were some discrepancies in the company's books, and that some funds seemed to be missing. Unbeknownst to Booker, Potsdam has been checking Booker's work after hours and he is convinced that Booker has been embezzling funds from Hyrax. Potsdam, on behalf of Hyrax, retains the services of Arnold Attorney to determine if Hyrax has a case against Booker. Arnold agrees with Potsdam that there is a strong indication that Booker has been embezzling funds. In fact, Arnold has already determined that tomorrow he will file a civil suit against Booker to recover Hyrax's money and go to the prosecutor's office to sign an embezzlement complaint against Booker. Arnold, however, tells Potsdam that the more evidence they can get against Booker, the stronger their case will be. Potsdam suggests that Arnold interview Booker before he presses charges, as Booker may make some remarks that would implicate him in the embezzlement. Arnold readily agrees to this. Potsdam tells Booker that Arnold is investigating some problems in account record keeping to make sure Hyrax's procedures comply with all applicable laws and regulations. He asks Booker to explain how his operation works. Arnold and Booker go to a private office, where Arnold interrogates Booker for approximately two hours. During the course of the interrogation, Booker becomes suspicious of the line of questioning, and asks if he is in any trouble. Arnold tells him not to worry, as the amount of money involved is so small that the action may not be worth pursuing. Is Arnold subject to discipline for questioning Booker? a. Yes, because all communications with parties who are not represented by counsel are prohibited. b. No, since Arnold has not yet filed suit on behalf of his client. c. Yes, because of his statements urging Booker not to worry. d. No, because Arnold was trying in good faith to further the interests of his client.

C.

Casper, a Hollywood movie producer who was charged under a criminal statute for unfair trade practices, now faces a civil claim under the same statute. Casper retains attorney Adams to represent him in both suits. Adams is a nationally known defense attorney who has represented many famous people. Most recently, he defended a celebrity in a notorious murder case that held the country rapt for several weeks. Adams explains to Casper that the representation is very complex and would take a majority of his time for several months. Given Adams' s steep hourly rate, Casper's legal fees would likely be around $1 million. Casper is short on cash and makes the following proposal: If Adams will represent him in both the civil and criminal suits, Casper will produce a movie based on Adams's most famous past cases, told from the lawyer's viewpoint. Adams would have complete creative control and would be entitled to all of the movie's profits, which could be anything from $0 to $100 million dollars. Casper had his personal attorney draw up a proposal to this effect, and Casper submitted it to Adams. Assuming that Adams receives any consent necessary from his former clients who might be portrayed in the movie, is this proposed arrangement proper? a. No, because any amount over $1 million is clearly excessive, and this arrangement could be worth $100 million. b. No, because a lawyer must not acquire media rights to a story concerning the lawyer's representation of a client. c. Yes, but only if the ultimate amount paid to Adams is not excessive in light of the work done. d. Yes, but only if the payment from the movie profits is for the civil suit only.

C.

Cesar conies to attorney Abigail with an invention he wants to patent. Cesar explains that he and a competitor have been racing one another to come up with the ideal cleaning solution. If Cesar's competitor were to find out that Cesar was at the patent stage, and worse, if he found out Cesar's formula, Cesar would be mined. Abigail, a trained and certified patent attorney, agrees to represent Cesar in the patent process. The invention involves complex chemical formulae, and Abigail's particular area of expertise is electronic devices. However, having worked with inventions of all types, she has no doubt that she can properly shepherd the solution through the patent process. In putting together the necessary paperwork, Abigail asks Barbara and David, who are associates ill her firm and hold chemistry degrees, to help her out on the project. In due time, Cesar's product receives a patent. Cesar's total bill for legal fees was $60,000, which was reasonable for the work done. When Abigail received Cesar's final payment, she decided to give Barbara and David each a $10,000 bonus from the fee. Are Abigail' s actions proper? a. No, unless Cesar consented after consultation to the disclosure of information to Barbara and David. b. Yes, unless the division of the fee is not in proportion to the work performed by Barbara and David. c. Yes, because with the aid of her associates, she was competent to handle the matter. d. No, unless Cesar consented to the splitting of the fee with Barbara and David.

C.

Client Celia comes to attorney Adnan seeking representation in bringing a breach of contract action. Celia tells Adnan that she originally retained lawyer Louis about a year ago, but that as far as she knew he had not even filed the papers. According to Celia, Louis never returned her calls, and when she went to his office to find out the status of her case, he was drunk and verbally abusive. Celia told her friends and family about Louis's treatment of her, and when her brother fared no better in getting information froth him, he suggested that she contact Adnan. Celia advised Adnan that since Louis did not return her calls, she had sent a certified letter to him notifying him that he was discharged. Adrian knew that there was only a one-year statute of limitations on this type of action, so he quickly checked the dates and discovered he had only a few days to file the action. Adnan called Louis to get tile information from Celia's file. Louis did not recall the letter of discharge and was surprised to get Adnan's call. He was, however, very cooperative and agreed to send a messenger to Adrian's office with Celia's file. Louis tells Adrian, 'I feel terrible about this. My wife has been very ill this year, and I haven't been myself. Tell Celia I am sorry. And I would appreciate it if you would keep this between the two of us. I would hate for this incident to scare away any more clients. 1 could sure use the money with my wife's illness and all." Attorney Adnan filed the papers in Celia's suit on time, and did not report Louis to the disciplinary authorities. Were Adnan's actions proper'? a. Yes, because it is Celia's decision whether to report Adnan to the disciplinary authorities. b. No, because he did not urge Celia to report Louis to the appropriate authorities. c. No, because Louis's actions indicate that he is not currently fit to practice law, d. Yes, because Adnan is an attorney, and Louis asked him to keep the matter confidential.

C.

Duffy graduated from law school, but he never took the bar examination and was never admitted to practice. He works as an investigator and paralegal for the law firm of Schnell & Gao, a professional corporation. Which of the following statements are true? I. On Duffy's recommendation, one of Duffy's friends retained lawyer Gao to represent her as plaintiff in a personal injury case. It would be proper for Schnell & Gao to pay Duffy 10% of its fee in the case as compensation for the referral. II. Lawyer Schnell frequently assigns Duffy to draft wills for Schnell's estate planning clients. Schnell supervises Duffy's work, revises Duffy's drafts, and is ultimately responsible for the final product. Schnell is subject to discipline for assisting a nonlawyer to engage in the unauthorized practice of law. III. Schnell & Gao established a retirement plan that is funded partly by legal fees earned by the Finn's lawyers. The firm may include Duffy as a beneficiary of the retirement plan. IV. Schnell & Gao established a stock option plan to compensate its personnel for hard work. The options allow recipients to purchase shares of stock in the law firm at a reduced price. It would be proper for Duffy to acquire stock in the firm through the stock option plan. a. I., II., III, and IV. b. III and IV. only. c. III. only. d. None of the above.

C.

For many years lawyer Lacy has done business transactions work for wealthy client Chung. Chung was recently injured in an automobile crash, and she has asked Lacy to represent her as plaintiff in an action against the driver who injured her. Lacy has taken some business cases to trial, but has never handled a personal injury case. Lacy would like to earn a profit from Chung's case. Which of the following would be proper ways for him to do so? I. Take the case and, with Chung's consent, associate a co-counsel who is competent in the field of personal injury law. II. Reier Chung to a competent personal injury lawyer and charge that lawyer a $1,000 forwarding fee. III. Take the case and do the study and re-search needed to handle it competently. IV. Refer Chung to a competent personal injury lawyer and charge Chung a reasonable sum for the time spent in making the referral. a. I. and IV. only. b. I., II., and III only. c. I., III., and IV. only. d. I. and III. only.

C.

For several years, attorney Aston worked at the United States Department of Labor as part of a small group of attorneys whose responsibilities included compiling certain corporate safety records and monitoring compliance with federal regulations. Under a federal statute, factories of a certain type and size must report all work related accidents to Aston's office. Aston's duties included compiling an annual report containing the accident statistics of all of the reporting companies. The report is used internally and in discussions with companies, but it is not distributed to the general public. A person may obtain a copy of the report, but must file a formal request under the Freedom of Information Act procedures adopted by the Labor Department. During the last three years, Chemco has bad more accidents than any of the other reporting companies. Six months ago, Aston left the Labor Department and took a job with a private law firm. Charles comes to Aston seeking representation in a suit against Chemco for injuries he sustained last month while working at one of Chemco's factories. Although unsure as to whether he should take the case, Aston, who is just starting out in private practice and cannot afford to turn clients away, agrees to represent Charles. Is Aston subject to discipline? a. Yes, because he obtained relevant information on Chemco while working as a government attorney. b. No, if Aston does not use the information obtained while employed as a government attorney to the material disadvantage of Chemco. c. No, because the information is available by formal request under the Freedom of Information Act. d. Yes, unless Aston obtains the consent of the Department of Labor.

C.

For the past 40 years, solo practitioner Febell has practiced municipal bond law in State A. Because he is nearing retirement age, Febell takes in young attorney Spryte as a partner. Their partnership agreement provides that Febell will train Spryte in municipal bond law, that Febell will receive 75% of the partnership's net earnings during the first three years, and that Spryte will receive the remaining 25%. The agreement further provides that if Spryte leaves the partnership before the end of the first three years, he will remit to Febell 75% of all tees he earns thereafter from municipal bond work he does in State A. Finally, the agreement provides that if Febell and Spryte are still partners when Febell retires, Spryte will pay Febell retirement benefits of $3,000 per month until Febell's death; in return, upon his retirement, Febell will turn over to Spryte all of the partnership assets (including good will) and will not thereafter practice municipal bond law in State A. Are Febell and Spryte subject to discipline for entering into this partnership agreement? a. No, because the agreement gives Febell retirement payments in return for the restriction on his right to practice. b. No, because the agreement enables Febell to sell the partnership assets in return for the restriction on his right to practice. c. Yes, because of the restriction on Spryte's right to practice if he leaves the partnership within the first three years. d. Yes, because of the restriction on both Febell's and Spryte's right to practice.

C.

For the past five years, attorney Aries has represented art dealer Corot in the sale of many valuable paintings. One of the major transactions occurred three years ago, when the American Museum of Art paid Corot $23 million for a Post-Impressionist landscape purportedly painted by Vincent Van Gogh in 1890. The American Museum of Art subsequently resold the painting to the Amsterdam Fine Arts Museum for $35 million. Today, Corot asked Aries to do the legal work in connection with the sale of a smaller, less valuable landscape, also a purported Van Gogh. The proposed purchase price is $12 million, and the prospective purchaser is Lavita Lavish, a wealthy television personality who knows nothing about art. During a confidential conversation in Arles's office, Arles said to Corot: "1 assume you have appraisal letters certifying the painting as a genuine Van Gogh?" Corot replied: "Of course I have letters! I forged them myself', just as I did for that bogus Van Gogh you helped me sell to the American Museum of Art three years ago!" When Aries inquired further, Corot told him in confidence that both of the purported Van Gogh paintings were in fact counterfeits created by a clever art student. Which of the following must Arles do at this point'? I. Report Corot to the law enforcement authorities. II. Warn Lavita Lavish about the proposed sale. III. Inform the American Museum of Art of the truth about the first painting. IV. Refuse to represent Corot in the present transaction. a. None of the above. b. 11., 111., and IV. only. c. IV. only. d. I., II., 111., and IV.

C.

Inventor Inovacio asked patent lawyer Patton to represent him in obtaining a U.S. patent on a new computer technique for predicting the growth patterns of tumors in the human body. Patton informed Inovacio that he had never worked on that kind of patent application before, and that he would have to do extensive background research on the patentability of computer techniques. Patton will be able to use the knowledge that he gains through the research to serve other clients who wish to obtain patents for all manner of other computer techniques. Patton offered to do the work for Inovacio for his standard hourly rate, but Inovacio proposed instead to assign Patton a 10% interest in the patent, if and when it was issued. Patton agreed to do the work on that basis, and he and Inovacio entered into an appropriate written fee agreement. Patton did the work; the patent was ultimately issued and proved so valuable that Patton was able to sell his 10% interest for $9.7 million. Is Patton subject to discipline? a. Yes, because he acquired a proprietary interest in the subject of the representation. b. Yes, because it is unreasonable to charge one client for background research that will be used to earn fees from other clients. c. No, unless $9.7 million is an unreasonably high tee for the work that Patton did. d. No, because Inovacio agreed to the fee arrangement.

C.

Judge Jacques is often referred to behind his back as "Judge Continuance" by lawyers who practice in Bayou County. This soubriquet is well deserved because Jacques is known to grant continuances whenever requested by an attorney, regardless of the substantiality of the attorney's grounds. He has turned down a continuance request on occasion, but such occasions are so few and fat' between that local attorneys are shocked when they hear of them. When queried by his colleague Judge Jeanne about his policy on continuances, Judge Jacques told her, "This society has gotten to be litigation crazy, it's sue, sue, sue, over every little thing. Most of this stuff can be settled between the parties if they really want to try. If I grant a continuance, it gives the parties that much more time to come to their senses and settle. If they settle, it saves them and the taxpayers the expense of a full-blown trial. This country and its court systems would be a lot saner if more judges tried to promote settlements like I do." Is Judge Jacques's policy of granting continuances to promote settlements proper? a. Yes, if Judge Jacques sincerely believes that his lenient continuance policy promotes settlements and that settlements promote a more rational and amicable system of justice. b. Yes, because the granting of continuances is clearly within the bounds of judicial discretion. c. No, because judges have a duty to expedite litigation. d. No, because judges have no duty to pro-mote settlements.

C.

Judge Jettelson sits on a United States Court of Appeals. He and two other Court of Appeals judges heard a diversity of citizenship case in which they were required to interpret a statute of State A concerning the marital communications privilege. Judge Jettelson's two colleagues wrote the majority opinion, in which they concluded that the statute gives only the witness-spouse the right to claim the privilege. Judge Jettelson wrote a vigorous and scholarly dissent, arguing that the statute gives both spouses the right to claim the privilege. Later, a State A senator legislator introduced a bill to amend the statute to reflect Judge Jettelson's position. The State Senate Justice Committee invited Judge Jettelson to testify about the public policy reasons for giving both spouses the right to claim the privilege. May Judge Jettelson testify? a. No, because a judge is not allowed to make public statements about disputed propositions of law, except when acting in his judicial capacity. b. Yes, but only if the two judges who wrote the majority opinion are also allowed to testify. c. Yes, because a judge may engage in activities designed to improve the law. d. No, because a judge must not become involved in politics, subject to certain exceptions that do not apply here.

C.

Judge is a judge of the trial court in City. Judge has served for many years as a director of a charitable organization that maintains a camp for disadvantaged children. The organization has never been involved in litigation. Judge has not received any compensation for her services. The charity has decided to sponsor a public testimonial dinner in Judge's honor. As part of the. occasion, the local bar association intends to commission and present to Judge her portrait at a cost of $4,000. The money to pay for the portrait will come from a "public testimonial fund" that will be raised by the City Bar Association from contributions of lawyers who are members of the association and who practice in the courts of City. Is it proper for Judge to accept the girl of the portrait? a. Yes, because Judge did not receive compensation for her services to the charitable organization. b. No, because the funds for the girl are contributed by lawyers who practice in the courts of City. c. Yes, because the gift is incident to a public testimonial for Judge. d. No, because the cost of the girl exceeds $1,000.

C.

Judge needed to obtain a loan to be secured by a second mortgage on his house. Bank offered him a loan at a very favorable interest rate. The vice-president at Bank told Judge: "Frankly, we normally don't give such a large loan when the security is a second mortgage, and your interest rate will be 2% less than we charge our other customers. But we know that your salary is inadequate, and we are giving you special consideration." Is it proper for Judge to accept the loan? a. No, unless the same terms are available to all judges in the state. b. Yes, if Judge does not act in any case involving Bank. c. No, because the amount of the loan and interest rate were not available to persons who were not judges. d. Yes, if Bank is not likely to be involved in litigation in the court on which Judge sits.

C.

Law Firm has 300 lawyers in 10 states. It has placed the supervision of all routine administrative and financial matters in the hands of Admin, a nonlawyer. Admin is paid a regular monthly salary and a year-end bonus of 1% of Law Firm's net income from fees. Organizationally, Admin reports to Attorney, who is the managing partner of Law Finn. Attorney deals with all issues related to Law Firm's supervision of the practice of law. Is it proper for Attorney to participate in Law Firm's use of Admin's services in this fashion? a. No, because Law Finn is assisting a nonlawyer in the unauthorized practice of law. b. Yes, unless Admin has access to client files. c. Yes, if Admin does not control the professional judgment of the lawyers in the firm. d. No, because Law Firm is sharing legal fees with a nonlawyer.

C.

Lawyer Lacey is on the in-house legal staff of Transcorp, Inc. In that capacity, she works daily with Transcorp's top executive officers. She was assigned to defend Transcorp in a lawsuit brought by West America Bank to collect a $750,000 promissory note. The note was signed on behalf' of Transcorp by Willard Westerman, Transcorp's treasurer and chief financial officer. Transcorp's defense is that Westerman had no authority to sign the note and that the bank knew it. Transcorp has advised Westernlan that it may seek indemnification from him if it is held liable to the bank. Westerman is not represented by counsel. Shortly before Westerman was to have his deposition taken by the bank, Westerman called Lacey and asked her what to expect at the deposition and how to respond to the bank's questions. What should Lacey do'? a. Tell Westerman that she cannot discuss the matter with him unless he wants her to represent him at the deposition. b. Advise Westerman that his own interests will be best served by answering truthfully and demonstrating, if he can, that he had authority to sign the note. c. Not discuss the matter with Westerman, and, if appropriate, advise him to hire a lawyer to represent him at the deposition. d. Advise Westerman to tell the truth, to answer fully all questions that are asked, and to pause before each answer to give her time to object to the bank's questions.

C.

Lawyer Larry and Client Connie have confidential communications concerning a partnership dispute. Litigation follows. Which of the following statements are correct according to the Arkansas Rules of Professional Conduct: (1) If Connie has died, then and only then Larry may disclose what Connie told him. (2) If Connie has died, then and only then Larry may disclose what he told Connie. (3) If Larry has died, then and only then Connie may disclose what Larry said to her. (4) If Larry has died, then and only then Connie may disclose what she told Larry. A) All four are correct. B) Only (1) and (2) are correct. C) Only (3) and (4) are correct. D) Only (1) and (3) are correct. E) Only (2) and (4) are correct.

C.

Lawyer Larry happened upon an accident scene and stopped his car to see if there was anything he could do to help. Several police officers were on the scene, and Larry told one of them that he was a lawyer and asked if he could do anything to assist the accident victims. The police officer told Larry, "One of the victims is a physician and he claims that he knows that he's going to die from his injuries and he keeps moaning about wanting a will." Larry went over to talk to the physician, who lay on a stretcher. The physician, Peter, begged Larry to write a will for him on the spot. Larry at first demurred, explaining to Peter that he had only been sworn into the state bar two weeks before and that he had never written a will for a client and had received a "D" in his only law school class covering the subject. Peter said, "I know the police officers will be glad to act as witnesses and I'll keep it simple; I just don't want those greedy worms in my mother's family to get their hands on my estate." After listening to five minutes more of Peter's pleading, Larry agreed to write the will for Peter. He wrote the will on the blank backside of an accident report. Peter signed the will and two police officers witnessed it. Larry told Peter, "Of course, there's no lee for this." Peter died two hours late,'. Was Larry's conduct proper? a. Yes, because Larry was a licensed attorney when he wrote the will. b. No, unless Peter agreed to limit Larry's malpractice liability. c. Yes, because he acted in a humane manner appropriate to an emergency situation. d. No, because Larry lacked sufficient knowledge of the law of wills.

C.

Lawyer Lawrence is a partner in the firm of Lawrence and Loeb. That firm regularly provides legal services to three major banks and two other important lending institutions in the community of Farmdale. Lawyer Lawrence has been invited to become a member of the board of directors of the Farmdale Legal Aid Society, the group that sets overall governing policies for the local Legal Aid office. One of the major issues that will soon face the board of directors is whether to amend the Case Intake Guidelines to allow the Legal Aid office to represent clients in disputes with banks and other lending agencies. Which of the following statements is most nearly correct? a. Lawrence should join the board of directors to help discharge her pro bono obligation, and she should vote in favor of amending the Case Intake Guidelines in order to make it easier for low income persons to sue banks and other lending institutions. b. Lawrence will be subject to discipline if she joins the board of directors, because service on the board is in conflict with the interests of her firm's bank and lending institution clients. c. Lawrence may join the board of directors, but she should refrain from participating in the decision about the Case Intake Guidelines. d. It would be proper for Lawrence to join the board of directors, and it would be proper for her to participate in the decision about the Case Intake Guidelines.

C.

Lawyer LeBrille is admitted to practice in State A. One of her regular clients is Chatsworth Inc., which is incorporated in and has its principal place of business in State A. The president of Chatsworth went to France to negotiate a business contract for Chatsworth that would be governed, in part, by the law of the European Community ("EC"). The president telephoned LeBrille to ask whether a particular provision of the proposed contract would be lawful under EC law. The president needed a quick answer because he had to resume the contract negotiation a few minutes later. LeBrille had studied EC law, but she was not admitted to practice in any nation that is a member of the EC. LeBrille warned the president about the danger of relying on off-the-cuff, unresearched legal advice, but he asked her to do the best she could. She then advised him that the contract provision would be lawful under EC law. The president thanked her, continued the contract negotiation, and signed a contract for Chatsworth that included the questioned provision. As it turned out, LeBrille's advice was mistaken: the provision violated EC law and rendered the contract unenforceable. Is LeBrille subject to discipline ? a. Yes, because she gave legal advice without adequate research. b. No, because a State A lawyer is not expect-ed to be competent in EC law. c. No, because she did the best she could in an emergency situation. d. Yes, because she is not licensed to practice EC law.

C.

Lawyer Loftand agreed to defend client Crow in a criminal case in which Crow was charged with arson. At Lofiand's request, Crow entrusted Loftand with his large diamond ring as a deposit to secure the payment of Lofland's fee. Loftand put the ring in her purse to take home to show her husband. When she took her car keys out of her purse, she unknowingly dropped the ring, and she was never able to find it. Crow was acquitted, paid Lofiand's fee, and asked for his ring back. At that point, Loftand told Crow what had happened. Which of the following statements is most nearly correct? a. Loftand is not subject to discipline, but she is subject to liability for malpractice. b. Loftand is not subject to discipline and she is not subject to liability for malpractice. c. Loftand is subject to discipline, and she is subject to liability for malpractice. d. Loftand is subject to discipline, but she is not subject to liability for malpractice.

C.

Lewis, an attorney, was formerly employed by the Environmental Protection Agency ("EPA") as government counsel in the New York office. in this capacity, Lewis acted as chief counsel in several suits brought by the EPA involving chemical dumping into public waterways. Two years after leaving the employ of the EPA, Lewis was retained to represent Acme Chemical Corporation in a suit brought by the EPA alleging violations of certain EPA regulations regarding the dumping of chemical wastes. While with the EPA, Lewis was never directly involved in a case concerning Acme Chemical Corporation. The EPA Washington office had exclusive responsibility for the drafting, promulgation, and enforcement of the regulations in question. Which of the following statements is correct? a. If Lewis represents Acme, he will be subject to discipline, even if the EPA consents to the representation. b. Lewis may represent Acme, but only with the consent of the EPA. c. Lewis may represent Acme, whether or not the EPA consents. d. If Lewis represents Acme without the consent of the EPA, he will be subject to discipline.

C.

Newshound regularly wrote the "Courts and the Law" column that appeared twice each week in the Daily Bugle, a newspaper of general circulation in the Bricktown area. Newshound approached Arlington, an attorney with offices in Bricktown. Newshound told Arlington that Arlington's name would appear frequently in Newshound's column in a favorable light, if Arlington would supply Newshound with "behind the scenes" items about local .judges, lawyers, and important or otherwise interesting cases. Arlington readily agreed to Newshound's proposal, and Arlington began supplying information to Newshound. Arlington's name did appear often in Newshound's column, and Arlington was characterized as a "top legal eagle," an "outstanding trial tactician," and otherwise touted in the newspaper. Is Arlington subject to discipline? a. Yes, because it is unethical to spy on fellow attorneys and judges. b. No, because Arlington's activities are protected by the First Amendment to the United States Constitution. c. Yes, because Arlington has given consideration for favorable publicity. d. No, because Arlington received nothing of pecuniary value from Newshound.

C.

Patricia Flores is a prosecutor who is prosecuting Alan for armed robbery. She has learned from law enforcement personnel that one of the three witnesses of the crime has recanted an earlier statement that identified Alan as the perpetrator. Which of the following statements most accurately describes Patricia's duties under the Arkansas Rules of Professional Conduct? A) Patricia must disclose to defense counsel the recantation if defense counsel makes a discovery request that reasonably could be construed to call for its revelation. B) Patricia must disclose to defense counsel the recantation if Patricia determines that this evidence more likely than not would result in acquittal of Alan. C) Patricia must disclose to defense counsel the recantation because this evidence tends to negate Alan's guilt. D) Patricia must disclose to defense counsel the recantation only if, without it, she no longer has probable cause to proceed with prosecution. E) Patricia need not disclose because the witness will be subject to cross-examination.

C.

Police officer Owens was charged with murder. He is alleged to have savagely beaten and ultimately killed a teenage gang member in the course of an arrest. Neither the police department nor officer Owens's union was willing to provide legal counsel for his defense, and Owens himself lacked funds to hire private counsel. The public defender's office could not represent him due to a conflict of interest from a related case. The trial court therefore appointed lawyer Lee to defend Owens. Lee is only three years out of law school. Lee practices criminal defense, but he has never handled a murder case before. For which of the following reasons may Lee decline the court appointment'? I. Based on what he has read in the newspapers, he sincerely believes that Owens is guilty. II. He has no experience in the defense of a murder case. III. He is of the same race as the teenage victim, and he is in sympathy with the plight of young gang members. IV. Many of his clients are of the same race as the teenage victim, and they will be irate if he defends Owens. a. I. and I1. only. b. II. only. c. None of the above. d. III. and IV. only.

C.

Pros, a prosecutor, was assigned to try a criminal case against Deft, who was charged with robbery of a convenience store. Deft denied any involvement, contending he was home watching television with his mother on the night in question. At the trial, Wit, a customer at the convenience store, testified that he had identified Deft in a police line-up and provided other testimony connecting Deft to the crime. In addition, Pros entered into evidence a poor-quality videotape of the robbery as recorded by the store surveillance camera. The jury convicted Deft of the crime charged. Unknown to Deft's court appointed lawyer, Wit had first identified another person in the police line-up and selected Deft only after encouragement by the detective. Pros was aware of these facts but did not notify Deft's counsel who made no pretrial discovery request to obtain this information. Is Pros subject to discipline? a. No, unless it is likely that the jury would have acquitted Deft had it known that Wit first identified someone else. b. Yes, unless the jury could make its own identification of Deft from the videotape. c. Yes, because this information tended to negate Deft's guilt. d. No, because Deft's counsel made no pretrial discovery request to obtain this information.

C.

Sales manager Morton testified before a federal grand jury that was investigating price fixing in me automobile tire industry. Ultimately the grand jury indicted Morton for price fixing, a felony under the Sherman Act. After his indictment, Morton sought to hire attorney Agnes to represent him at his criminal trial. Morton is a middle class business executive with enough savings to pay for private counsel. He told Agnes in confidence that he had lied to the grand jury about several meetings he had had with competitors. Further, he told her that he wanted to plead not guilty to the criminal charge, and he told her that he intended to testify as he did before the grand jury. Which of the following would be proper for Agnes to do in this situation? I. Decline to represent Morton. II.. Agree to represent Morton, and represent him as best she can at the criminal trial, III. Inform Morton that unless he pleads guilty to the criminal charge, she will tell the prosecutor about his false testimony before the grand jury. IV. Decline to represent Morton, and inform the prosecutor about his false testimony before the grand jury. a. IV. only. b. I. or II. only. c. I. only. d. I. or III. only.

C.

The legislature of State A conducted open hearings concerning a bill pending in the state legislature that would make it much more difficult for corporations chartered in State A to be taken over by corporate raiders. By making unfriendly takeovers more difficult, the proponents of the bill hoped to save jobs in State A and to encourage corporations not now chartered in State A to obtain State A charters which would bring added revenue to the state. Attorney Armor, a senior partner in the prestigious firm of Armor, Baldwin, and Chase, asked to testify at the hearings. Armor had been retained by Tentacle Corporation and was asked by Tentacle's president to testify against the pending legislation at the legislative hearings. Tentacle's president also told Armor that under no circumstances was he to tell the legislature that he was working for Tentacle or to mention the name of Tentacle in his testimony. Armor complied with Tentacle's strictures and never mentioned that he was being retained to give testimony against the pending legislation. Armor's testimony before the legislative committee was effective and hard-hitting. After his testimony, Armor was asked a few questions by committee members, but he was never asked if he was appearing on some other party's behalf. Was Armor's conduct at the hearing proper? a. No, because an attorney may not practice deception upon a legislative body. b. Yes. because no one asked Armor if he was appearing at the hearings on behalf of a client. c. No, because he did not disclose that he was appearing at the hearings in a representative capacity. d. Yes, because his client specifically instruct-ed him not to reveal the client's name.

C.

The state bar association has offered Judge and her spouse free transportation and lodging to attend its institute on judicial reform. Judge is expected to deliver a banquet speech. Is it proper for Judge to accept this offer? a. No, if members of the bar association regularly appear in Judge's court. b. Yes, unless the value of the transportation and lodging exceeds $500. c. Yes, because the activity is devoted to the improvement of law. d. No, because the bar association is offering free transportation to Judge's spouse.

C.

Three years ago, lawyer Lenoski successfully defended Wolner in a narcotics case. In the course of that work, Lenoski obtained significant confidential information about Wolner's life history. Now the local trial court has appointed Lenoski to defend Drexel in a totally unrelated narcotics case. The prosecution's key witness against Drexel is Wolner. If Lenoski accepts the appointment, will she be subject to discipline? a. No, because she would be representing Drexel under court appointment. b. No, because the two narcotics cases are unrelated to each other. c. Yes, if the confidential information she learned from Wolner would be relevant to impeach Wolner's testimony against Drexel. d. Yes, unless Drexel is fully informed of the situation and gives his written consent.

C.

Two years ago, Attorney was employed by State's Department of Transportation (DOT) to search title to several tracts of land. Attorney has not been employed by DOT during the last year. Recently, DOT instituted proceedings to condemn a tract, owned by Owner, for a new highway route. Owner asked Attorney to represent her in obtaining the highest amount of compensation for the condemnation. Owner's tract is one of the tracts on which Attorney searched title two years ago. Attorney remembers that Engineer, a DOT engineer, once drafted a confidential memorandum advising against running a new highway across Owner's land because of potential adverse environmental impact. Because of this information, Attorney believes it is possible to prevent the condemnation of Owner's land or to increase the settlement amount. It is proper for Attorney to: a. Represent Owner on the issue of damages only and not disclose the information that might prevent the condemnation. b. Represent Owner and attempt to prevent the condemnation by using the information about the adverse environmental impact. c. Refuse to represent Owner and not disclose the information about the adverse environmental impact. d. Refuse to represent Owner but disclose to Owner the information about the adverse environmental impact.

C.

United Consumers Bank operates a "Professional Referral Hotline" for its depositors. Any United depositor who needs to find a physician, lawyer, accountant, dentist, or the like, can telephone the hotline and obtain a free referral from lists of professionals compiled by United. The lists are limited to professionals who maintain an average balance of at least $10,000 in a time deposit account at United, but the professional does not pay a fee to United for receiving a particular referral. Lawyer Lomax keeps $10,000 on deposit with United for the express purpose of being included on its lawyer referral list. Is this arrangement proper? a. Yes, because United is functioning in the role of a lawyer referral service. b. No, because this arrangement constitutes an association with a nonlawyer for the practice of law. c. No, because Lomax is required to keep $10,000 on deposit to be included on the list. d. Yes, because neither United's depositors nor the professionals pay a fee for referrals.

C.

When Slocum was a third-year law student, she worked as an extern for the State Commission on Energy Resources, an administrative agency that regulates gas and electric utilities throughout the state. As an extern, she worked personally and substantially on the application of Geron Gas, Inc., for a certificate of convenience and necessity that would allow Geron to build a gas pipeline from the northern to the southern part of the state. Three months after Slocum's externship ended, the Commission denied Geron's application. After Slocum graduated from law school, she became an associate in the private firm of Lewis & Levy. Geron asked partner Levy to represent it in taking a court appeal from the Commission's denial of its application. Must Levy decline the case? Yes, because Slocum worked on the case personally and substantially. No, provided that Slocum does not herself assist him on the case. Yes, unless Slocum is screened from the case, shares no part of the fee, and the Commission is notified in writing. No, because the Commission denied the application after Slocum's externship ended.

C.

Alfonso is mayor of the city of Dustbowl. Alfonso is also a licensed attorney who has a law partnership with Bella. Under the city charter of Dustbowl, the mayor has the authority to determine what issues are to be placed upon the agenda of the city council. Several council members have told Alfonso that they would like to see a particular zoning measure placed upon the agenda. This proposed ordinance would ban commercial development of a certain area within the city limits. Bella has been retained as attorney for Octopus Development Corp. Octopus has acquired land in the proposed noncommercial zoning area and has plans to construct a large shopping center there. May Bella represent Octopus in this matter? a. Yes, because Octopus is not a client of Alfonso. b. Yes, if Alfonso is not present at any city council meetings at which the matter is discussed. c. No, unless Alfonso has no direct role in the representation and does not share in any fees from the case. d. No, because of Alfonso's position as mayor.

D.

While she was attending law school at night, Adela served as a clerk for Judge Jones of the county court. During the course of her employment by Judge Jones, the case of Lyndon v. Baines came before Jones. The case was very complicated and Adela did a lot of research on the case for Jones, submitting a number of memoranda on issues in the case. Jones always carefully supervised his clerks, and this case was no exception. He was exceptionally pleased with the care and high quality of Adela's work. Shortly after Jones handed down his final judgment on Lyndon v. Baines, Adela was admitted to the bar and she accepted employment with Nixon, Dixon, and Yates, a prominent local law firm. A few days later, Baines, who was the loser in the Lyndon v. Baines case, appeared at the offices of Nixon, Dixon, and Yates. He told the interviewing attorney, "My lawyer at the trial was terrible. I hired him because he was cheap, and I guess I got what 1 paid for. I'd like your firm to handle the appeal." If Nixon, Dixon, and Yates accepts Baines as a client, and Adela's supervising attorney asks Adela to handle the appeal, would it be proper for Adela to do so? a. Yes, because Adela was not licensed to practice law when she worked on the case for the judge. b. Yes, if no confidential information was revealed to Adela during her work on the case for the judge. c. No, because Adela was previously personally and substantially involved in work on the case. d. No, unless Judge Jones consents.

C.

XYZ law firm defended Wal-Mart in a slip and fall accident in Fort Smith in 2016. Ann Associate was with the firm at the time, but did not work on the case. Ann left XYZ in 2017. Ann is now with ABC law firm. On behalf of Victim Vinny, ABC has now sued Wal-Mart for a slip and fall accident that occurred in Fayetteville in 2021. Ann will not be working on the case. XYZ will be defending Wal-Mart. Do the Arkansas Rules of Professional Conduct permit ABC to represent the plaintiff against Wal-Mart? Can ABC ethically represent the plaintiff against Wal-Mart? A) Yes. There is no ethical issue because the three year cooling off period has been satisfied and any conflicts have lapsed. B) Yes, provided ABC builds a Chinese wall and screens Ann from the litigation. C) Maybe. The burden rests on ABC (or Ann) to overcome the rebuttable presumption. D) Maybe. The burden rests on XYZ (or Wal-Mart) to overcome the rebuttable presumption. E) No. ABC hired a tainted lawyer. Under the rules of imputed disqualification, the firm is barred from representing Vinny. The presumption is irrebutable.

C.

You are representing Amber the Accuser, the plaintiff in a civil lawsuit. Amber says she loaned Ramon $5,000 which Ramon now refuses to pay. You have become suspicious that Amber is not telling you the complete truth. You also suspect that she may lie when you call her as a witness tomorrow. You counsel her, and you tell her that lying on the stand is a criminal act; she merely says "Thank you for your advice." Under the governing ethical standards, which actions are you now permitted to take? Consider the three options. 1) Do nothing. Put Amber on the stand and treat her as a normal witness. 2) Notify opposing counsel that Amber may lie tomorrow when she is on the stand. 3) Do not allow Amber to testify. A) Either 1 or 2 is permitted. B) Either 2 or 3 is permitted. C) Either 1 or 3 is permitted. D) Only (3) is permitted E) Any of the three options are permitted.

C.

Alexandra is a recent law school graduate who has just been admitted to the State Bar. She returns to her hometown, Sodville, a town of 20,000 in population in the center of the state. Gastrix, who has practiced out of a one person office in Sodville for many years, asks Alexandra to associate with him. He produces an employment contract that he asks Alexandra to sign. Gastrix is the only attorney in Sodville who regularly handles bankruptcy cases. The following provisions in the employment contract would bar Alexandra, for one year after leaving Gastrix's employ, from: I. Practicing law within a 50-mile radius of Sodville. II. Accepting any cases dealing with bankruptcy. III. Accepting business from clients who had been represented by Gastrix's law firm during the period of Alexandra's employment. For agreeing to which of the above provisions would Alexandra be subject to discipline? a. III. only. b. I. and II., but not III. c. I. only. d. I., II., and III.

D.

Attorney Abrahamson recently opened his solo law practice in Crystal Springs. His practice is fairly evenly divided between civil litigation and criminal defense. The Crystal Springs Superior Court has just appointed him to represent defendants Denton and Drews, who will be tried jointly for their alleged kidnapping and brutal murder of nine Crystal Springs school children. For which of the following reasons may Abrahamson decline the appointment? I. He believes that to represent Denton and Drews will take so much time away from his newly opened practice as to impose an unreasonable financial burden on him. 11. He believes that Denton coerced Drews into helping kidnap and kill the children. III. He believes many of his potential clients in Crystal Springs will be outraged if he defends Denton and Drews. IV. He believes that confidential information he received when representing one of the prosecution's key witnesses will be useful in impeaching that witness's credibility. a. II. only. b. I., II., III., and IV. c. II. and IV. only. d. I., II., and IV. only.

D.

Attorney Alex has a high-profile divorce law practice in Centerville. Because of his heavy case load, Alex often appears before the four chancery judges of the county court. One of the chancery judges, Jamal, is getting married, and he sends a wedding invitation to Alex. Alex wishes to send Jamal, as a wedding gift, an imported Italian machine that makes espresso and cappuccino coffee because Alex knows that Jamal loves fine coffee. The coffee machine sells for $200 at Centerville's best cooking equipment store. Would it be proper for Alex to send the coffee machine to Jamal? No, because the gift is not a campaign contribution, and lawyers should not give other types of gifts to judges. Yes, unless Jamal believes that he would be unduly influenced by the gift. No, because the value of the gift exceeds $1500. Yes, but only if Alex and Jamal are friends and only if the gift is not excessive for the occasion and the relationship.

D.

Attorney Allison, a member of the state bar, and Judge Jack, who sits on the district court, set up "The Profitable Probate School" ("PPS"). The announced purpose of the school was to educate laypersons as to probate procedures, to enable them to file their own papers and avoid the expenses of obtaining counsel for the probate process. The tuition was a modest $50 per student, which barely covered the expenses of books and forms handed out to the students. Despite the "Profitable" in PPS's name, Allison and Jack donated their time and received no remuneration from the school. However, each student was required to sign a paper which stated, "The $50 tuition fee establishes an attorney-client relationship between an attorney teaching at PPS and all students. PPS and attorneys teaching therein are not liable for any damages that might be incurred by students as a result of pro se filings made in accordance with or at variance from instructions given by PPS." The statement was signed by Allison and Jack as "officers of the school." Is Allison subject to discipline? No, because the clients agreed to the contract as a precondition for obtaining the benefits offered by PPS. No, because Allison is helping the public to avoid excessive legal fees through her participation in the school. Yes, because she has participated in the forcing of an attorney-client relationship upon the students. Yes, because an attorney may not limit her malpractice liability by contract.

D.

Attorney Alpha filed a complaint on behalf of Client against Agri, a corporation, alleging that Agri had breached a valid oral contract entered into on Agri's behalf by Pres, the president and chief executive officer of Agri, to sell Client certain merchandise for a specified price. Attorney Beta, representing Agri, has filed an Answer denying the contract and asserting the statute of frauds as a defense. Attorney Beta has given notice to Alpha that he will take the deposition of Pres on the grounds that Pres will be out of the country on the date the case is set for trial. Pres is not a shareholder of Agri. Alpha would like to interview Pres, prior to the taking of the deposition, in order better to prepare her cross-examination. Is Alpha subject to discipline if she interviews Pres without Beta's knowledge and consent? a. Yes, because Pres is being called as an adverse witness. b. No, because Pres allegedly entered into the contract on behalf of Agri. c. No, unless Pres will be personally liable to Agri for damages in the event judgment is rendered against Agri. d. Yes, because Pres is the president of Agri, who is represented by counsel.

D.

Attorney Alpha is recognized as an expert in securities regulation law. Corp, a corporation, retained Alpha's law firm to qualify Corp's stock for public sale. After accepting the matter, Alpha decided that he preferred to spend his time on cases with larger fee potential, so he assigned responsibility for the Corp matter to Attorney Beta, an associate in Alpha's office who had recently been admitted to the bar. Beta protested to Alpha that he, Beta, knew nothing about securities regulation law and that he had too little time to prepare himself to handle the Corp matter competently without substantial help from Alpha. Alpha responded, "I don't have time to help you. Everyone has to start somewhere." Alpha directed Beta to proceed. Was Alpha's conduct proper in this matter? a. No, because Corp had not given Alpha permission to assign Beta to work on the matter. b. Yes, because as a member of the bar, Beta is licensed to handle any legal matter. c. Yes, because Alpha may withdraw from a case if work on it would cause him unreasonable financial hardship. d. No, because Alpha knew Beta was not competent to handle the matter, and Alpha failed to provide supervision adequate to protect the client's interest.

D.

Attorney represented Landlord in a variety of matters over several years. Plaint, an elderly widow living on public assistance, filed suit against Landlord alleging that Landlord withheld, without justification, the security deposit on a rental unit that Plaint vacated three years ago. She brought the action for herself, without counsel, in small claims court. Attorney investigated the claim and learned that it was legally barred by the applicable statute of limitations, although Plaint's underlying claim was meritorious. Attorney told Landlord of the legal defense, but emphasized that Plaintiffs claim was just and that, in all fairness, the security deposit should be returned to Plaint. Attorney told Landlord: "I strongly recommend that you pay Plaint the full amount with interest. It is against your long-term business interests to be known in the community as a landlord who routinely withholds security deposits even though the tenant leaves the apartment in good condition. Paying the claim now will prevent future headaches for you." Was Attorney's conduct proper? a. Yes, if Landlord did not object to Attorney's advice and paid Plaint's claim. b. No, unless Attorney's engagement letter informed Landlord that Attorney's advice on the matter would include both legal and non-legal considerations. c. No, because in advising Landlord to pay the full claim, Attorney failed to represent zealously Landlord's legal interests. d. Yes, because Attorney may refer to both legal and non-legal considerations in advising a client.

D.

Attorney Augusta decided to run against incumbent Judge Isadore in the forthcoming election. Isadore was widely regarded by members of the local bar as a "party hack," who had no business being on the bench. The opposition party was very pleased to be able to slate Augusta because she had a high reputation for intelligence, honesty, and overall competence as an attorney. Augusta realizes that she will have to fight an uphill battle to unseat Isadore because her political party is a minority party in the county and most voters know very little about judges and candidates for the judiciary and therefore, voters are likely to vote a straight ticket for judges of their own political party. Augusta wants the public to know that Isadore has been a poor judge, but she also wishes to comply with all ethical rules governing judicial campaigns. Attorney Eve, Augusta's best friend and chief advisor, suggests that Augusta should make the following statements during her campaign: I. "Judge Isadore has had the highest percentage of cases reversed on appeal of any judge in the state over the past two years." II. "Eighteen months ago Judge Isadore was publicly disciplined by the State Judicial Conduct Board." III. "A recent poll taken by the local bar association indicates that a majority of bar association members feel that Judge Isadore lacks the proper judicial temperament." IV. "A recent newspaper article comparing judges of the county states that Judge Isadore has handed out an average sentence of only two and a half years to persons convicted of serious felonies. I won't be soft on crime!" Assume that all the facts cited in the numbered statements are accurate. Which, if any, of the numbered statements would it be proper for Augusta to make in her judicial campaign'? a. III. and IV., but not I. and II. b. I. and II.. but not III. and IV. c. II. and III., but not I. and IV. d. I., II., and III., but not IV.

D.

Attorney Axelson has been retained by Sludge Corporation. Axelson has done legal work for Sludge for many years and is on very friendly terms with its officers and directors. Axelson is also Chair of the State Bar Association's Committee on Corporate Law. A bill has been introduced into the state legislature that would allow corporate boards of directors to vote by telephone, thus eliminating the necessity for the directors to be physically present at meetings. The State Bar Committee on Corporate Law has been studying the bill and is about to have a meeting where a vote will be taken on whether to recommend to the legislature that the bill be passed. The committee's recommendation will probably carry a great deal of weight with the legislators, and in fact may well be determinative of whether the bill is enacted into law. Axelson meets the president and the treasurer of Sludge after work lot cocktails at the Snob Club, a private club to which Axelson and the officers of Sludge belong. Although the occasion is primarily social, the president of Sludge tells Axelson that he is very interested in the work of the State Bar Committee on Corporate Law. The president tells Axelson that he and the other officers of Sludge strongly favor the bill pending in the legislature, as telephone voting would be much more efficient for Sludge and would save all the time and trouble of gathering the directors together/hr meetings. He urges Axelson to argue in favor of the bill in the committee meeting and to vote in favor of recommending that the bill pass. Is it proper for Axelson to support the corporation bill in the State Bar Committee? a. No, if his vote will be decisive in determining the Bar Committee's recommendation. b. Yes, since he will be acting to further the interests of a client. c. No, since he has a conflict of interest. d. Yes, if he conscientiously believes the bill is in the public interest and if he discloses to the committee that one of his corporate clients will benefit if the bill passes.

D.

Attorney Sarah worked for 3 years in the tax section of ABC, a major firm which has been involved in the defense of nursing home claims. Sarah left the firm and has now practiced by herself for approximately 18 months. She has now been offered a job with a plaintiff's firm. The firm specializes in nursing home cases and has numerous ongoing cases against clients of ABC. Many of them have been in litigation for more than 3 years. A) If Sarah joins the firm both she and the firm will be disqualified from all pending nursing home cases against ABC. B) If Sarah joins the firm, she will be disqualified from all pending nursing home cases against ABC, but the firm will not be. C) If Sarah joins the firm, neither she nor the firm will be disqualified because she left ABC more than 12 months ago, which is after the "cooling-off" period. D) If Sarah joins the firm, the presumption is that she and the firm are disqualified. The presumption is rebuttable, and the burden is on Sarah and the firm. E) The presumption is that she and the firm have no conflict of interest. The burden is on ABC to rebut the presumption.

D.

Attorney is a well-known tax lawyer and author. During congressional heatings on tax reform, Attorney testified to her personal belief and expert opinion on the pending reform package. She failed to disclose in her testimony that she was being paid by a private client for her appearance. In her testimony, Attorney took the position favored by her client, but the position was one that Attorney believed was in the public interest. Was it proper for Attorney to present this testimony without identifying her private client? a. Yes, because Attorney conscientiously believed that the position she advocated before Congress was in the public interest. b. Yes, because Congress is interested in the content of testimony and not who is paying the witness. c. No, because a lawyer may not accept a fee for trying to influence legislative action. d. No, because a lawyer who appears in a legislative hearing should identify the capacity in which the lawyer appears.

D.

Attorney represented Husband and Wife in the purchase of a business financed by contributions from their respective separate funds. The business was jointly operated by Husband and Wife after acquisition. After several years, a dispute arose over the management of the business. Husband and Wife sought Attorney's advice, and the matter was settled on the basis of an agreement drawn by Attorney and signed by Husband and Wife. Later, Wife asked Attorney to represent her in litigation against Husband based on the claim that Husband was guilty of fraud and misrepresentation in the negotiations for the prior settlement agreement. Is it proper for Attorney to represent Wife in this matter? a. No, unless Husband is now represented by independent counsel. b. Yes, if all information relevant to the litigation was received by Attorney in the presence of both Husband and Wife. c. Yes, if there is reason to believe Husband misled both Wife and Attorney at the time of the prior agreement. d. No, because Attorney had previously acted for both parties in reaching the agreement now in dispute.

D.

Attorney represents Client, a well-known contractor, before Agency, a state administrative agency. Agency has ordered Client to show cause why Client's license as a contractor should not be revoked for violation of agency regulations. In a newspaper interview prior to the administrative hearing, Attorney truthfully stated that: I. "Client denies the charge made by Agency that Client engaged in conduct constituting grounds for revocation of Client's license as a contractor." II. "The next step in the administrative process is the administrative hearing; if Agency is successful, we will appeal, and Agency still cannot revoke Client's license until a court affirms the finding for Agency." III. "Client needs witnesses who are aware of the incidents that are the subject of the hearing." Which of these statements would be proper? a. I only b. II only c. Ill only d. I, II, and III

D.

Attorney represents Client, the plaintiff in a civil action that was filed a year ago and is about to be set for trial. Client informed Attorney that he could be available at any time during the months of October, November, and December. In discussing possible trial dates with opposing counsel and the court clerk, Attorney was advised that a trial date on October 5 was available and that the next available trial date would be December 10. Without first consulting Client, Attorney requested the December 10 trial date because she was representing Deft, the defendant in a felony criminal trial that was set for October 20 and she wanted as much time as possible to prepare for that trial. Was it proper for Attorney to agree to the December trial date without obtaining Client's consent? a. Yes, because a criminal trial takes precedence over a civil trial. b. No, because Attorney should manage her calendar so that her cases can be tried promptly. c. No, unless Attorney was court appointed counsel in the criminal case. d. Yes, unless Client will be prejudiced by the delay.

D.

Attorney represents Driver, the plaintiff in an automobile accident case. Two weeks before the date set for trial, Attorney discovered that Wilt was an eyewitness to the accident. Attorney interviewed Witt. Witt's version of the accident was contrary to that of Driver and, if believed by the trier of fact, would establish that Driver was at fault. Witt told Attorney that she had not been interviewed by defense counsel. Witt also told Attorney that she intended to leave for Europe the following week for a one-month vacation unless she had an obligation to remain and attend the trial. Attorney told Witt: "No one has subpoenaed you yet. You have no legal duty to make yourself available. Trials can be difficult affairs. Witnesses sometimes get very nervous because of the questions asked by the lawyers. Why don't you take the vacation as planned, and, by the time you return, the trial will be over." Is Attorney subject to discipline.'? a. Yes, because Attorney did not subpoena Witt knowing she was an eyewitness. b. Yes, because Attorney advised Witt to leave the jurisdiction. c. No, because Attorney did not offer Witt any inducement not to appear at the trial. d. No, because Witt had not been subpoenaed by the defense.

D.

Attorney was employed as a lawyer by the state Environmental Control Commission (ECC) for ten years. During the last two years of her employment, Attorney spent most of her time in the preparation, trial, and appeal of a case involving the discharge by Deftco of industrial effluent into a river in the state. The judgment in the case, which is now final, contained a finding of a continuing and knowing discharge of a dangerous substance into a major stream by Deftco and assessed a penalty of $25,000. The governing statute also provides for private actions for damages by persons injured by the discharge of the effluent. Attorney recently left the employment of ECC and went into private practice. Three landowners have brought private damage actions against Deftco. They claim their truck farms were contaminated because they irrigated them with water that contained effluent from dangerous chemicals discharged by Deftco. Deftco has asked Attorney to represent it in defense of the three pending actions. Is Attorney subject to discipline if she represents Deftco in these actions? a. Yes, unless the judgment in the prior case is determinative of Deftco's liability, b. No, because Attorney has acquired special competence in the matter. c. No, if all information acquired by Attorney while representing ECC is now a matter of public record. d. Yes, because Attorney had substantial responsibility in the matter while employed by ECC.

D.

Attorney, who had represented Testator for many years, prepared Testator's will and acted as one of the two subscribing witnesses to its execution. Testator's sister and brother were his sole heirs. The will left Testator's entire estate to his sister and nothing to his brother. Upon Testator's death two years later, Executor, the executor named in the will, asked Attorney to act as his lawyer in the probate of the will and the administration of the estate. At that time, Executor informed Attorney that Testator's brother would concede that the will was properly executed but intended to contest the will on the ground that he had been excluded because of fraud previously practiced on Testator by Testator's sister. The other subscribing witness to the will predeceased Testator, and Attorney will be called as a witness solely for the purpose of establishing the due execution of the will. Is it proper for Attorney to accept the representation? a. No, unless Attorney's services are necessary to avoid substantial hardship to Executor. b. No, because Attorney will be called as a witness in the case. c. Yes, because Executor has no beneficial interest under the will. d. Yes, if there is no contested issue of fact with respect to the formal execution of the will.

D.

Attorney, who is corporate counsel for Company, is investigating a possible theft ring in the parts department of Company. Attorney knows that Employee has worked in the parts department for a long time and believes that Employee is a suspect in the thefts. Attorney believes that if Employee were questioned, Employee would not answer truthfully if she knew the real purpose of the questions. Attorney plans to question Employee and falsely tell her that she is not a suspect and that her answers to the questions will be held in confidence. Is Attorney subject to discipline if she so questions Employee? a. Yes, unless Attorney first advises Employee to obtain counsel to represent Employee. b. No, because Attorney did not give legal advice to Employee. c. No, because no legal proceedings are now pending. d. Yes, because Attorney's conduct involves misrepresentation.

D.

Author Arthur wrote a best-selling novel based on the life and crimes of John Dillinger, the famous bank robber. Arthur sold the movie rights to film producer Prosser, who promised to pay Arthur a lump-sum royalty of $5 million upon release of the movie. After Prosser hired actor Clint Nickleman to play the lead role and made other expensive preparations for filming, Arthur repudiated the contract. Prosser hired lawyer Laine to sue Arthur for a declaratory judgment that the contract was valid and enforceable. At Prosser's request, Laine agreed to do the legal work on a contingent fee basis: If Prosser wins, Laine will be paid 1.75% of the gross receipts from the movie, but if Prosset loses, Laine will be paid nothing. Prosser and Laine entered into a written fee agreement that contains all the details required by the rules of legal ethics. Which of the following statements is true? a. Laine is subject to discipline for acquiring a personal interest in the subject of the litigation. b. Laine's fee agreement is proper, but only if Arthur consents after full disclosure. c. Laine is subject to discipline for entering into a publication rights contract with his client. d. Laine's fee agreement is proper, even though it gives Laine a personal interest in the subject of the litigation.

D.

Barbara Barrister is in private practice. She represents the mother in a contested divorce and custody battle. Outside the presence of her client, she interviews the 8 year old boy Tommy: When Mommy is away, Daddy drinks and gets mad and hurts me. Select the correct answer. 1) Under the Rules of Professional Conduct, Barbara is obligated to report this information to the appropriate authorities. 2) Under the Rules of Professional Conduct, Barbara may report this information to the appropriate authorities. 3) Under the child abuse reporting statute, Barbara is obligated to report this information to the appropriate authorities. 4) Under the child abuse reporting statute, Barbara may report this information to the appropriate authorities. A) Both 1 and 3 are correct. B) Both 1 and 4 are correct. C) Both 2 and 3 are correct D) Both 2 and 4 are correct E) Without the permission of Tommy's mother, the attorney can not disclose the conversation.

D.

Capra hired lawyer Laslo to represent him in a personal injury action against Giant Co. As Capra and Laslo discussed the case, Capra stated, "Of course, I can't predict what the future will hold, but I am ready to go all the way on this. If we have to go to trial to see justice done, so be it. I probably wouldn't agree to a settlement under $500,000, no matter what." Laslo agreed that the claim was worth at least that, but felt they would receive a much higher award if the case went to a jury. Shortly before the trial started, the lawyer for Giant Co. contacted Laslo with a settlement offer of $150,000. Laslo tried to call Capra, but couldn't reach him. After two hours of trying to reach Capra, Laslo called Giant Co. back and rejected the offer. At trial, the jury awarded Capra $1 million. Is Laslo subject to malpractice liability for his actions? a. Yes, because decisions to accept or reject settlement offers arc to be made by the client. b. Yes, because a lawyer has a duty to keep his client informed of all settlement offers. c. No, because Capra impliedly authorized Laslo to reject any offer under $500,000. d. No, because the jury award was much greater than the settlement offer

D.

Castor and Pollux are law partners. Castor represents Scurvy, and Pollux represents Smythe. Both are criminal defendants whose cases are seemingly unrelated. During the course of an interview with Castor, Scurvy tells him that he was involved in the crime with which Smythe is charged and that he is willing to testify against Smythe if he can be granted immunity from prosecution on that charge and plea bargain the crime with which he is presently charged down to a lesser offense. Which of the following courses of action is proper for Castor? a. Inform Pollux of what Scurvy has told him and continue to represent Scurvy. b. Inform Pollux of what Scurvy has told him and withdraw from representing Scurvy. c. Not inform Pollux and continue representation. d. Not inform Pollux and withdraw from representation.

D.

Client, a new client of Attorney, has asked Attorney to write a letter recommending Client's nephew for admission to the bar. Client has told Attorney that he has no direct contact with the nephew, but that Client's sister (the nephew's mother) has assured Client that the nephew is industrious and honest. Which of the following would be proper for Attorney? I. Write the letter on the basis of Client's assurance. II. Write the letter on the basis of Client's assurance if Attorney has no unfavorable information about the nephew. III. Make an independent investigation and write the letter only if Attorney is thereafter satisfied that the nephew is qualified. a. I and II, but not III b. I, II, and lII c. I and III, but not II d. III only

D.

Deborah, who herself had been the subject of sexual discrimination, represented an employee in a civil sexual discrimination case against her employer. In closing arguments, Deborah told the jury, "I believe that sexual discrimination is rampant in this country and should be eliminated. If ever there was a just cause, my client's case is it." Is it proper for the attorney to make this statement? A) Yes, because, in a civil matter, an attorney may state her personal opinion as to the culpability of the defendant. B) Yes, because a lawyer is required to zealously represent her client. C) No, because the matter is one of personal interest to the attorney. D) No, because a lawyer, in addressing the jury, may not give her personal opinion as to the justness of a cause. E) Yes, because Deborah has personal knowledge.

D.

Delta, a lawyer, has just joined the Law Offices of Alpha and Beta, a professional corporation engaged solely in the practice of law. Delta is a salaried associate and is not a member or shareholder of the professional corporation. Alpha's spouse, Veep, who is not a lawyer, is vice-president of the corporation and office manager. All of the other officers are lawyers in the firm. Alt of the corporate shares are held by lawyers in the corporation, except for ten shams held by the executor under the will of a lawyer member who died one month previously and whose will is now being probated. Delta knows that Veep is an officer and not a lawyer. Is Delta subject to discipline? a. Yes, if a non-lawyer holds the stock as the executor of the will of the deceased member. b. No, because Delta is a salaried employee and not a member or shareholder of the corporation. c. No, if Veep does not participate in any decision regarding a client or a client's case. d. Yes, because Veep is an officer of the corporation.

D.

Diplock, a juvenile, has been charged with auto theft, a felony, and has hired Lucinda to represent him. Before the case comes to trial, Lucinda confers with Patrick, who has been assigned by the district attorney's office to prosecute the case. Patrick knows that Lucinda plans to run for public office and is always interested in getting as much publicity as possible. She is often referred to in the press as a "hotshot" criminal defense attorney, and almost all of her victories over the state are publicized. Patrick suggests to Lucinda that, since Diplock is a juvenile, the charge might be reduced to "joyriding," a misdemeanor. Lucinda refuses, telling Patrick, "I win by trying cases. You must not have read the newspapers lately--there's going to be a lot of coverage of tiffs ease because Diplock's father is socially prominent, so I'm not about to cop a joyriding plea for tile kid." If Diplock is convicted of auto theft, would it be proper for Patrick to report Lucinda to the disciplinary authorities? a. Yes, but only if he is certain that Lucinda's motive was to gain publicity. b. No, unless Lucinda got publicity from the trial. c. No, if Lucinda defended Diplock competently at trial. d. Yes, if he knows that Lucinda never conveyed the offer of a lesser charge to Diplock.

D.

During the course of researching a reply brief, attorney Amy discovered a case in the controlling jurisdiction that seemed to be right on one of the key issues involved in the litigation that she wa~3 researching. Although much of the dicta in the case seemed to favor Amy's client, one critical sentence in the holding clearly put the court behind the position asserted by Amy's opponent, Bertram. However, Bertram had done a slipshod job of researching the issues and had failed to find the case and to cite it in his brief. Amy decided to cite the case in her reply brief, but she cited the case as favoring her client by quoting much of the dicta and deliberately omitting the key sentence in the holding. Amy filed the reply brief with the court and sent a copy to Bertram, who she knew often bragged, "I haven't actually read a case since my first year of law school." Amy was also aware that this judge's clerks tended to be overworked and so did not always read all the cases cited by attorneys in their briefs. Therefore, she hoped that the negative aspects of the case might slip by unnoticed by the judge and opposing counsel. Is Amy subject to discipline? a. No, because an attorney has a duty to present cases in the light most favorable to her client. b. No, because an attorney has no duty to educate, and Amy fulfilled her duties to the court and opposing party by citing the case. c. Yes, because an attorney has a duty to cite all opposing cases accurately and objectively. d. Yes, because Amy is attempting to mislead the tribunal.

D.

During the trial of a case by attorney Alpha, attorney Beta, who had been playing a round of golf at Country Club, takes refuge from a sudden rainstorm in the "19th Hole Pub," Country Club's tavern for members and their guests. While in the "19th Hole," Beta sees attorney Alpha seated at a corner booth having a drink with Snyrd, whom Beta knows to be a juror in the case Alpha is presently trying. Attorney Beta considers his options; he can: I. Keep this knowledge confidential. II. Identify himself to Alpha and reprimand Alpha for drinking in a public place with a juror. III. Inform the State Bar. What should Beta do? a. II. only. b. II. and III. only. c. 1. only. d. III. only.

D.

Judge Jackman is a full-time judge in State A. Her father lives in a retirement home in State B. Judge Jackman's father told her on the telephone that several of his friends in the retirement home had employed attorney Abbott to write wills for them, that in each will attorney Abbott had included a bequest to himself, and that each bequest was approximately .5% of the estimated total value of the person's probable estate. The friends told Judge Jackman's father that they did not really want to leave Abbott anything; they had assumed it was merely a matter of routine, a part of the lawyer's compensation for drafting the will. Attorney Abbott is admitted to practice in State B, but not in State A. Judge Jackman did not talk personally with any of her father's friends, but she believes that her father's rendition of the story is entirely accurate. Would it be proper for Judge Jackman to communicate directly with attorney Abbott about the matter, and if that does not satisfy her, to communicate with the attorney disciplinary authority in State B about the matter'? a. No, because she is not allowed to communicate directly with Abbott about the supposed legal ethics violation. b. No, because legal ethics violations that take place outside State A are not her concern. c. Yes, because she has personal knowledge that Abbott has violated a legal ethics role. d. Yes, because she has received information indicating a substantial likelihood that Abbott has violated a legal ethics rule.

D.

Judge Josephine ruled in favor of Plaintiff in a civil action where Defendant was ordered to pay Plaintiff $50,000 in damages. Josephine has since resigned from the bench. Defendant has refused to pay the $50,000, asserting that the verdict was obtained through improper means. Defendant asks Josephine, now in private practice, if she will represent him. Would Josephine be subject to discipline if she represents Defendant? a. No, unless Josephine was a party to fraud when the original verdict was handed down. b. Yes, because former judges may not engage in private practice. c. No, because Josephine is no longer on the bench. d. Yes, because Josephine ruled on this case when she was a judge.

D.

Judge is one of three trustees of a trust for the educational benefit of her grandchildren. The trust owns 5,000 shares of stock in Big Oil Company. The stock has been selling for the past year at $10 per share. Big Oil is suing Oil Refining Company for breach of an oil refining agreement, and the case is assigned to Judge for trial. Judge believes that she can be fair and impartial. Must Judge disqualify herself from the case? a. Yes, unless the outcome of the lawsuit is unlikely to affect the value of the stock. b. No, because Judge believes she can remain impartial. c. No, unless Judge personally owns stock in either party to the litigation. d. Yes, because the trust has more than a de minimus financial interest in Big Oil Company.

D.

Law Firm, a professional corporation with five lawyer shareholders, employs twenty-five additional lawyers. Which of the following is (are) proper? I. Employees who are members of the bar are not made shareholders until they have been with Law Firm ten years. 1I. Manager, who is the office manager but not a member of the bar, is executive vice president of Law Firm. III. Widow, whose husband was a lawyer shareholder in Law Firm until his death two years ago, continues to hold husband's shares in Law Firm, distributed in his estate, until their child completes a law school education. a. I, II, and III b. I and II, but not III c. I and III, but not II d. I only

D.

Lawyer Leavitt practices environmental law. He also happens to be one of the nation's leading experts on the environmental effects of filling wetlands. The legislature of State A has scheduled hearings on a bill to prohibit the filling of wetlands surrounding Clearwater Bay. One of Leavitt's regular clients is Bay View Development Company, which owns development rights to some of the wetlands in question. Bay View wants to fill its wetlands so that it can build low-cost housing for underprivileged families. Bay View hired Leavitt to appear as a witness at the legislative hearings and to testify in opposition to the ban on wetland filling. Leavitt appeared as a witness, identified himself as an expert on wetlands, and testified vigorously against the proposed legislation. Was Leavitt s conduct proper? a. No, because a lawyer must not be a witness for his client on a contested matter. b. Yes, unless his testimony was contrary to his own beliefs about the environmental effects of filling wetlands. c. Yes, because he is a leading expert on the environmental effects of filling wetlands. d. No, unless he informed the legislators that he was appearing in a representative capacity.

D.

Lawyer Li represented Weaver in a court proceeding to raise the alimony and child support payments set in the decree that divorced Weaver from her ex-husband, Hyde. Hyde stubbornly refused to get a lawyer in the marten The evening before the court hearing, Hyde telephoned Li at home and asked Li to explain the legal standard the judge would apply to Weaver's request for increased payments. Li responded: Mr. Hyde, I am not your lawyer, and I cannot give you legal advice. I think that you ought to get a lawyer in this matter, and if you need time to do that, I will ask the judge to postpone the hearing for a couple of weeks. Hyde said he didn't want a lawyer, and then asked Li whether Weaver and the children really needed more money to live on. Li responded: Mr. Hyde, I have no personal interest to serve here--I am simply trying to do what is best for you, and your ex-wile, and your kids. Now if you really want my opinion, I'd say yes, you should pay the extra money because they do need it to live on. Hyde thanked Li and hung up. Was Li's handling of the matter proper? a. No, because as Weaver's lawyer, Li should not have communicated directly with Hyde at all. b. Yes, because Li only stated his opinion and did not purport to give Hyde advice. c. Yes, because Li advised Hyde that Li was not his lawyer, that Hyde should retain one, and that Li could not give him legal advice. d. No, because Li pretended to be disinterested and advised Hyde to pay the extra money.

D.

Lawyer Locaro represents patent owner ProData Corp. as plaintiff in a multimillion dollar patent infringement action against defendant Datatec, Inc. In searching ProData's files for documents responsive to a Datatec demand for production of documents, Locaro found some old invoices that are clearly called for in Datatec's demand. The invoices are ambiguous. By one interpretation, they are totally irrelevant to the case. By another interpretation, they will prove that ProData's patent is invalid and unenforceable. Like all well-run organizations, ProData has a routine "File Maintenance Program," which calls for throwing away outdated documents. But for a clerical error by a lazy employee, the old invoices would have been tossed out years ago. When Locaro found the invoices, he told ProData's chief executive officer what he had found. The CEO said: "Throw them away." What should Locaro do? a. Throw away the invoices. The law regards as done that which ought to have been done. But for the lazy employee's error, the invoices would have been thrown away long ago; thus, Locaro should do what ought to have been done. b. Either throw away the invoices or withdraw. As an advocate, Locaro should abide by the decisions of his client on all matters that affect the merits of the case. Here, the client speaks through its CEO. Locaro should either follow the CEO's instructions or withdraw. c. Throw away the invoices. As an advocate, Locaro should resolve factual ambiguities in favor of his client. By the interpretation that favors his client, the invoices are irrelevant and do not have to be produced. d. Either produce the invoices or withdraw. If the CEO refuses to produce the invoices, Locaro should raise the matter with ProData's board of directors; if they too refuse, Locaro should withdraw.

D.

Lawyer Lohman (age 34) regularly represented client Cruikshank (age 78) in matters relating to the investment of Cruikshank's considerable wealth. Cruikshank told Lohman that he wanted to put $500,000 into a sound, income-producing investment. Lohman suggested that the two of them pool their money and talent and buy an attractive new apartment house. Lohman would put up $75,000 and do the legal work, and Cruikshank would put up $500,000 and serve as the live-in manager of the apartment house. Cruikshank enthusiastically agreed to the arrangement and told Lohman to draw up the papers. Lohman drafted an agreement between himself and Cruikshank, negotiated the purchase of the apartment house, and drafted a deed from the seller to himself and Cruikshank as joint tenants with right of survivorship. Lohman gave Cruikshank a carefully written explanation of the terms of the transaction, but he forgot to explain the significance of the joint tenancy, i.e., that upon the death of one joint tenant, the property would pass automatically to the other joint tenant. Lohman urged Cruikshank to have an outside lawyer look over the transaction, but Cruikshank said he trusted Lohman and signed all the papers without further ado. Lohman and Cruikshank operated the apartment house successfully for several years, until Cruikshank died at age 83. The executor of Cruikshank's estate sued Lohman to have the apartment house declared part of Cruikshank's estate, but the court concluded that the joint tenancy created a gift to Lohman, effective on Cruikshank's death. Were Lohman' s actions proper? a. Yes, because Lohman might have died first, thus bestowing a gift on Cruikshank. b. No, because Lohman entered into a business transaction with Cruikshank. c. Yes, because the court concluded that the joint tenancy created a gift from Cruikshank to Lohman. d. No, because Lohman drafted the deed that bestowed a substantial gift on himself.

D.

Lawyer Lorenz agreed to represent wife Withers on an hourly fee basis in securing a divorce from her husband Hullar. Hullar is represented in the matter by attorney Atwell. Despite repeated warnings by Lorenz, Withers kept pestering Lorenz with telephone calls and office visits concerning inconsequential details and trifling personal complaints. When Withers was unable to contact Lorenz on the phone or in person, she would telephone Atwell, her husband's attorney, and try to put her questions and complaints to him. Atwell always refused to talk to Withers. Lorenz repeatedly told Withers not to contact Atwell, but to no avail. Finally, Lorenz told Withers that she would withdraw unless Withers changed her ways, but Withers did not do so. Lorenz withdrew and sent Withers a fee bill for the total number of hours she had spent on the case. Withers refused to pay the bill, and after futile efforts to settle the matter, Lorenz sued her to collect the fee. Which of the following propositions are true? I. It was proper for Lorenz to withdraw. II. It was proper for Atwell to refuse to talk with Withers on the phone. III. It was proper for Lorenz to bill Withers for the total amount of time she spent on the case. IV. It was proper for Lorenz to sue Withers to collect the unpaid fee. a. I. and III. only. b. None of the above. c. II. only. d. I., II., III., and IV.

D.

Lawyer Lydia represents defendant Dave, who is being prosecuted in a jury trial for an armed robbery and attempted murder that occurred on June 15. Dave has pleaded not guilty to the charges, but Lydia knows that Dave is the perpetrator and that the crime occurred at approximately 10 p.m. Vivian, the victim, testifies that she is certain that the crime occurred at 12 a.m. midnight. Dave has an airtight alibi for midnight. At 11:40 p.m. he was arrested on a drunk driving charge, and he was in police custody until 6 a.m. on June 16. On cross-examination, Lydia does nothing to challenge Vivian's recollection of the time of the attack. Also, as the trial unfolds, Lydia does not introduce ally evidence at her disposal that would help establish the time of the attack at 10 p.m. Lydia calls as a witness Officer Tatum, who testifies that Dave was in fact in custody at midnight on the night in question. Dave does not testify and is acquitted. Are Lydia's actions proper';' a. No, because she knew that Vivian's testimony was wrong and would mislead the jury as to a crucial component of the case. b. No, unless she notified tile judge of the true facts outside the presence of the jury, and he instructed her to proceed. c. Yes, because Lydia's client is a criminal defendant and constitutional protections take precedence over ethical roles. d. Yes, because Lydia did not present false evidence

D.

Paralegal Platen works for the law firm of Dahlers & Sentz. Her direct supervisor is partner Dahlers, whose practice is limited to international trade law. Partner Sentz is the firm's leading trial lawyer, both in commercial and personal injury cases. On her way to work one morning, Platten saw a pedestrian run down in a crosswalk by a speeding car. Platten rendered first aid, and while she was waiting with the pedestrian for the ambulance, Plattcn said: "Here, call the number on this card and talk to attorney Seymour Sentz; he's really good, and he can help you recover money for the injury you have suffered." When she got to work, she told partner Dahlers what she had done. Dahlers admonished Platten not to hand out the firm's cards in such situations, but he did not discuss the matter with partner Sentz. Is Dahlers subject to discipline? a. No, because as a nonlawyer, Platten is free to recommend a lawyer to someone if she wishes. b. Yes, because as Platten's supervisor, he is responsible for any unethical act she commits. c. No, because Platen may not have been aware of the time that she did anything wrong. d. Yes, because he failed to warn Sentz not to take the pedestrian's case.

D.

Paul is a title insurance agent. In serving his customers, he routinely fills in the blanks in standard form documents that are prepared by lawyers. These documents include warranty deeds, quitclaim deeds, mortgages, releases of mortgages, affidavits as to debts and liens, lien waivers, and the like. On occasion, when his customers specifically ask, he advises them about the meaning and legal effect of the technical language used in the forms. Which of the following constitutes the unauthorized practice of law by Paul? I. Filling in the blanks on warranty deeds, quitclaim deeds, and mortgages. II. Filling in the blanks on affidavits as to debts and liens and lien waivers. III. Advising customers, at their request, about the meaning and legal effect of the technical language used in the forms. a. I. only. b. None of the above. c. III. only. d. All of the above.

D.

Probate attorney Adamson was representing the executor of decedent Denman's estate. The executor removed the contents of Denman's safe deposit box and brought them to Adamson to be inventoried and appraised. The items included Denman's collection of valuable antique gold coins. Adamson put the coin collection into a heavy brown envelope and labeled it as part of the Denman estate. Because he intended to start preparing the inventory immediately after lunch, Adamson put the brown envelope and Denman's other belongings into the file drawer of his desk; he then left for lunch without locking the file drawer. Adamson's secretary saw the coins and saw what Adamson did with them. During the lunch hour, the secretary took the envelope of coins and disappeared, never to be seen again. Is Adamson subject to discipline'? a. No, because the loss was proximately caused by the secretary's dishonesty, not by his conduct. b. Yes, because he is responsible for his employee's dishonest act. c. No, because he took reasonable precautions to safeguard the coins in the circumstances. d. Yes, because he did not put the coins in a sate place.

D.

Shortly after the county grand jury handed down an indictment for armed robbery against fugitive Drooles, District Attorney Daly met with the working press outside the door to the grand jury room. Daly told the press, "You all know that I'm limited as to what I can say about pending cases, so I only have three statements to make to you. You can save your time and mine by not asking any other questions because 'no comment' is the only way I'll be able to answer them." Daly made the following three statements to the press: I. "Drooles has been indicted by the grand jury for armed robbery, but like all other American citizens he should be considered innocent until proven guilty." II. "Drooles was indicted after grand July testimony by two credible witnesses." III. "The public should be warned that Drooles is a fugitive and is considered to be armed and dangerous." Which of the above numbered statements was proper for the District Attorney to make? a. I., II., and III. b. I. and II. only. c. I. only. d. I. and III. only.

D.

Solo practitioners Arias and Armer share office space. Each of them has organized her practice as a professional corporation. The sign on their office door reads: Arlene Arias, PC. Personal Injury Law/Alice Armer, PC General Practice. Arias and Armer frequently consult with each other about their respective cases, and they often refer clients to one another. Sometimes they work on cases together under a fee-sharing arrangement. When one of them is out of the office, the other responds to client inquiries to the extent that she is able; to facilitate that practice, each attorney has physical access to the other's client files. Plaintiff Puente hired Arias to sue McDougal's Bakery for personal injuries he sustained when he bit into a piece of' glass in a dinner roll baked by McDougal's. McDougal's liability insurance carrier, American Assurance Associates, asked Armer to serve as defense counsel in the case. May Armer take the case ? a. No, unless Arias and Armer believe that they can effectively represent their respective clients. b. Yes, but only if Arias and Armer believe that they can effectively represent their respective clients, and only if Puente, McDougal's, and American consent after full disclosure. c. Yes, because the role of imputed disqualification does not apply to Arias and Armer. d. No, even if Arias and Armer believe that they can effectively represent their respective clients, and even if Puente, McDougal's, and American consent after lull disclosure.

D.

Stork, Inc. is a large private adoption agency that handles over 65% of all private adoptions in State Blue. Stork provides each set of adoptive parents with a list entitled "Approved List of Lawyers Handling Private Adoptions." Stork's representatives tell prospective adoptive parents that it is in their best interest to obtain counsel who has experience in adoptions. Stork has checked out every lawyer on its list to make sure that the lawyer had experience in private adoptions as well as a reputation for honesty and ethical behavior. Because of Stork's volume of business, State Blue attorneys recognize the advantages of being placed on the "Approved List." After an attorney desiring placement on the list has been screened by Stork, the attorney is required by Stork to sign a form agreement before the lawyer's name is placed on the list. To assure the adoptive parents that any lawyer on the list would follow through with the adoption to its conclusion, the required form agreement contains the statement, "1 agree that under no circumstances will 1 withdraw from any case where I have been retained by parents adopting through Stork." Attorney Ash has handled many private adoptions and is highly regarded as an honest and competent attorney. He would like to be placed on Stork's list. Would it be proper for Ash to have himself included in the list? a. Yes, because the Approved List is a valuable service, and people often do not know where to find a lawyer qualified to handle a matter such as a private adoption. b. Yes, provided Ash does not give anything of value to Stork in exchange for being placed on the list. c. No, because Stork is acting as Ash's agent, and a lawyer may not use an agent to contact prospective clients in a manner that would be unethical solicitation if done by the lawyer. d. No, because the form agreement allows a third party to exercise influence over the lawyer-client relationship.

D.

The Bar of State A has established a peer counseling program whereby lawyers who are addicted to alcohol or other drugs can receive confidential counseling from other lawyers. The Bar of State A's ethics rule on confidential information provides that communications between the counselor lawyer and the counseled lawyer are to be treated just like confidential communications between attorney and client. Lawyer Loontis is addicted to alcohol and is receiving peer counseling under the program from lawyer Lin. Loomis is a large, strong man, and his addiction has made him subject to periodic fits of physical violence. This afternoon, during their peer counseling session, Loontis told Lin: "My client Crothers has refused to pay the fees he owes me; the next time I get drunk enough, I'm going to smash the little creep's face in." From working with Loomis over an extended period, Lin believes that he may really do it. May Lin disclose Loomis's statement to Crothers, the police, and the Bar of State A? a. No, unless Loomis consents. b. No, unless Lin is certain that Loomis will carry out his threat. c. Yes, because Lin is serving as a peer counselor, not a lawyer. d. Yes, even if Loomis objects.

D.

The State Bar and the State University are joint sponsors of the State Continuing Legal Education Foundation. The purpose of the Foundation is to provide continuing legal education to lawyers and judges in the state. Its board of directors is composed one-half of members of the legal profession and one-half of university personnel. Judge Anistopolous has been invited to serve on the board of directors. Which of the following are correct? I. She may serve on the board of directors, as long as it does not interfere with her judicial duties. II. It would not be proper for her to serve on the board of directors, since to do so would involve her in the teaching of law. III. If she serves on the board of directors, it would not be proper for her to accept the modest salary that is paid to other directors. IV. If she serves on the board of directors, she may accept the modest salary that is paid to other directors. a. II. only. b. None of the above. c. I. and III. only. d. I. and IV. only.

D.

The law firm of Rice and O'Malley represents P.J. Kilmer, a successful businesspers0n who has given the firm almost all of his substantial legal business. Kilmer is presently involved in complex civil litigation in which Kilmer stands to receive a large damages award if the suit is successful. Harmon, the attorney for the defendant in the case, has filed a motion that, if granted, would result in dismissal of the suit. The named partners have studied tile motion and feel that there is some merit in the motion, and at least a 50% chance that Judge Jensen, who is trying the case, will rule in favor of the defendant. During the course of explaining the defendant's motion to Kilmer, Rice and O'Malley call in Costello, who is a recently hired associate. Costello is Judge Jensen's former law clerk, and wrote several speeches for her when the judge was running in a contested reelection campaign. Rice and O'Malley tell Kilmer that it would be to his advantage for Costello to argue against the motion. Costello, during the course of the meeting with the client, tells Kilmer, "I think we'll get a break on this because Judge Jensen owes me a favor." Is Costello subject to discipline? a. Yes, if Costello plans to use his knowledge of Judge Jensen's character to his advantage. b. No, if Costello honestly believes that he will do the best job of arguing the motion on its merits. c. No, unless Costello plans to use improper influence on the judge. d. Yes, since his statement implies that the judge will give him preferential treatment because of their past association.

D.

Tom Tuttle is; the trustee of a trust for the care and support of his deceased sister's minor children. Tuttle wishes to sell some of the trust property to pay for the schooling of one of the children, who has special needs. Tuttle hires lawyer Lemke to file the appropriate papers to get court approval for the sale. In the course of the conversations between Lemke and Tuttle, Tuttle discloses that he has committed several breaches of trust in the past, including borrowing trust funds to pay for his home improvements and gambling trust funds at the race track. Most of the money has been repaid with reason able interest, and Tuttle tells Lemke that he has learned a few things about being a trustee, and will be much more careful about his handling of trust funds in the future. Lemke urges Tuttle to tell the court of his wrongdoing and resign as trustee, but Tuttle refuses. Lemke proceeds to represent Tuttle in the proceeding seeking court approval for the sale of trust assets. Lemke fills out all of the court papers truthfully and does not in any way slate anything false or misleading lo the court. The court does not inquire about the management of the trust or any dissipation of trust assets, and neither Lemke nor Tuttle volunteer the information. Is Lemke subject to discipline? a. Yes, because he could have prevented future fraud by Tuttle. b. No, because it is the court's duty to supervise a trustee. c. Yes, because he owes a duty of candor to the tribunal. d. No, because tile information was confidential.

D.

When lawyer Locke was an associate in the firm of Bliss & Buford, she did the legal work for one of the firm's clients, Cannon, on a land sale transaction that earned Cannon millions of dollars. In gratitude, Cannon asked Locke whether she had any unfulfilled wishes. Locke told him that she wished she had enough money to start her own solo ]aw practice. Cannon then told her that he would lend her $100,000 to set up her new practice. In return, she would thereafter do all of his legal work at a 5% discount from her normal hourly fee, and she would pay Cannon 10% of the legal fees she earned in solo practice until the $100,000 loan was fully repaid. Locke was delighted. She drafted a complete, detailed agreement between herself and Cannon, and she insisted that he get outside legal advice before signing the agreement. Cannon got the outside advice and signed the agreement, and Locke set up her solo practice accordingly. Is Locke subject to discipline? a. Yes, unless Bliss & Buford consented to the loss of Cannon as a firm client. b. No, unless she allows Cannon to interfere with her professional judgment in handling work for other clients. c. No, unless the terms of the transaction were unfair to Cannon. d. Yes, unless Cannon is a lawyer.

D.

While an assistant district attorney, Attorney Alpha was in charge of the presentation before a grand jury of evidence that led to an indictment charging thirty-two defendants with conspiracy to sell controlled drugs. Shortly after the grand jury returned the indictments, Alpha resigned as assistant district attorney and became an associate in the law office of Attorney Beta, a sole practitioner. At the time of such association, Beta was the attorney for Deft, one of the indicted co-defendants. Is it proper for Attorney Beta to continue to represent Deft? a. Yes, if Alpha does not reveal to Beta any confidence or secret leaned while an assistant district attorney. b. Yes, because a public prosecutor must make timely disclosure to the defense attorney of any exculpatory evidence. c. No, unless Alpha agrees not to participate in the representation of Deft. d. No, because Alpha had substantial responsibility for the indictment of Deft.

D.

Worker Workman sued his employer, Drexel Moving and Storage Co., claiming that he was permanently and totally disabled due to a back injury he suffered on the job. Lawyer Lenhart represented Drexel in the case. Lenhart strongly suspected, but had no proof, that Workman continued his hobby of skydiving after the alleged back injury. In due course, Lenhart met with Workman's lawyer for a settlement discussion. Lenhart told Workman's Lawyer: "We won't give you a dime on this claim; we've got movies of your guy jumping out of an airplane two weeks after his phoney injury." Workman's lawyer excused herself to make a telephone call to Workman. When she asked Workmail whether he had been skydiving after the accident, he admitted that he had. With the consent of their' respective clients, the two lawyers then settled the case for $400. Is Lenhart subject to discipline? a. No, because Lenhart's bluff successfully unmasked a fraudulent claim. b. Yes, because it was improper to pay $400 to settle a fraudulent claim. c. No, because bluffing is an accepted tactic in settlement negotiations between lawyers. d. Yes, because Lenhart lied about having movies.

D.

Young associate Aster was assisting senior partner Parker in writing the reply brief in an appeal for one of Parker's clients. In doing the legal research, Aster discovered a recent case from the controlling jurisdiction that had not been cited in the adversary's brief. In Aster's opinion, the case was directly opposed to the position of Parker's client. Aster asked Parker about citing it in the reply brief, but Parker explained that, in his view, the case was not directly on point and did not have to be cited. Aster and Parker argued back and forth at some length and finally decided to submit the question to one of the other senior partners in the firm for a fresh view. That partner sided with Parker, and the reply brief was filed without mentioning the case. Should Aster write a short letter to the appellate court and the adversary lawyer, explaining his position and enclosing a copy of the case? a. No, because Aster must not communicate with a court ex parte about the merits of a pending case. b. Yes, because Aster must not allow another person to interfere with his professional judgment. c. Yes, because Aster had a duty to call the case to the court's attention. d. No, because Aster should abide by Parker's resolution of the matter.

D.


Related study sets

Chapter 11 Confessions and Admissions: Miranda v. Arizona

View Set

Property and Casualty Insurance Practice

View Set

Entrepreneurship and small business test 3

View Set